You are on page 1of 104

rddf"i, $qil\a.

i1lq-

ACE
Engineering Academy
HyderabadlDelhilBhopallPunelBhubaneswarlBengalurulLucknowlPatnalChennailVijryawadrlVisakhapahamlTirupatilGunturlKukatpally(IIyd)

Steel Structures
Volume - I: Study Material with Classroom Practice Questions

GATE r Civil Engineering


ACE is the leading institute.for coaching in IES, GATE & PSUs
H O: 204,II Floor, Rahman Plaza, Opp. Methodist School, Abids, Hyderabad-s0OOOl,
Pbz 04A-23234418,040-23234419, 040-23234420;040 - 24750437 : , ,

Consistently Top Ranks In trES


32 AII India 1* Ranks in GATE
01 Materials and Specifications 01-06
02 Riveted and Bolted Connections 07 -23
03 Welded Connections 24-36
o4 Eccentric Connections 37 -46
05 Tension Members 47 -57

06 Compression Members 58-69


07 Column Bases and Column Splices 70 -76
08 Beams 77 -88
09 Plate Girders 89-95
10 Gantry Girders 96-98
TL RoofTrusses 99 - 105
Materials and Sp ecifications
teel structures
Steel str are built up with hot - rolled steel 3. IS 2062: (Fusion welding type):
sections
ections . Steel commonly used for general
ger
. Hot - rolled steel sections are made up of construction pulpose, particularly suil
;uitable
structural steel for structure subjected to dynamic: loads
l,
. IS 800-2007: code of practice for use of and impact such as bridge decking, girders
gir
structural steel in general building and crane girders.
construction . Designated as Fe410-WA,Fe4l0-' 0-wB,
. Types of structural steel: Fe4l0-WC.
. IS 226 (Standard Quality)
. Suitable for welding in all thickness.
. IS 2062 (Fusion welding Quality)
. Carbon content 020%-0.25%
. IS 961 (High tensile steel)
. Elongation:23o/o
. IS 1977 (Ordinary Quality) 4. IS 961 (High tcnsile steel):
. IS 8500 (Medium & high strength . Greater strcngth and atmospt
atmospheric
qualities)
corrosion resistance
.
2. lS 226 (Standard Quality): . Fe 570 HT: For strucrurercture with
. Most commonly used steel for general fabrication by methods other than fusion
fu
construction purposes of
buildings, welding.
bridges, industrial structures, transmission . Fe 540 - W - HT: For structures,tures where
w
line towers etc. fusion welding is involved.
. Riveting, bolting can be done for all . Carbon content : 0.21 o/o for Fe
le 570 HT
-
thickness but welding is permitted for . Elongation: 20o/o
thickness < 20mm only
. Carbon content:0.23 to 0.25o/o, Elongation:23o/o
. Designated as : Fe 410 - S

Yield strength (fr) (MPa)

UTS Elongation
Type of steel Grade / Gauge
(MPa) Thickness (mm)
Classification
(f") 5.6sJsb
<20 2040 >40

Standard Structural steel E 250 (Fe 4104) 4t0 250 240 230 23
(Standard Quality steel IS 226 & E 250 (Fe 4l0B) 410 250 240 230 23
Fusion welding Quality IS 2062) E 250 (Fe 4l0C) 410 250 240 230 23

<16 r6-40 4r-63


Fe 440 440 300 290 280 22
Micro alloyed high strength steel
Fe 540 540 410 390 380 20
rs 8500
Fe 590 s90 450 430 420 20

HydenbadlDelhilBhopal.fPunelBhulaneswlBensatm[_IdgknowfPannlChemifvijapqadalVizag
lTf+UCFJeqpffl fufaqd]y(Ilyd)
Jfr.
:IDjf
ACE
ngineerins academv 2 Steel Strucnues

5. Physical properties ofstructural steel: a) Rolled steel I - sections or Beam


sections:
Phvsical Proper8 IS 800
s =7.85
o Indian Standard Junior Beam [SJB)
Soecific sravitv
Unit mass of steel o. = 7850 Ke/m' a Indian Standard Light Beam (ISLB)
Modulus of Elasticity
- E,:2 x l0'N/mm' a Indian Standard Medium weight Beam
p:0.2s (IS800:1984)
Poisson's ratio (ISMB)
u = 0.30 [5800:2007)
G:0.8 x l0'N/ mm' . Indian Standard Wide flange Beam
(IS800:1984) (ISWB)
Modulus of Rigidity
G:0.769 x lOsN/mm2 . Indian Standard Heavy Beam (ISHB)
0S800:2007)
Coefficient of thermal exPansion a.: 12 x 10-6 /"C . Indian Column Section (SC)

6. Various types of rolled structural steel


. Section is designated by its depth
An I -
and weight
section:
a) Rolled steel I-sections (beam section) Eg: An ISLB 500 atl35.8 N/m means, An l-
b) Rolled steel channel sections section is 500 mm deep and self weight is
c) Rolled steel Tee sections 735.8 N per meter length.
d) Rolled steel angle sections b) Rolled Steel Channel Sections:
e) Rolled steel bars . Indian Standard Junior Channel (ISJC)
0 Rolled steel flats . Indian Standard Light Channel [SLC)
g) Rolled steel Plates
h) Rolled steel sheets . Indian Standard Medium Weight Channel
i) Rolled steel strips with Sloping Flange (MC)
j) Rolled steel tubular sections . Indian Standard Medium Weight Channel
with parallel flange (MCP)
. Indian Standard Gate Channel (ISGC)
. Designated by its depth and weight
Eg: ISLC 350 at 380.63 N/m
c) Rolled Steel T - Sections:

Rolhd b€amB tsolled ch*nltsle An$lgd (un€+uat)


. Indian Standard rolled Normal
ard €olumil3
T - section (ISNT)
. Indian Standard rolled Deep legged
r - (ISDr)
Indian Standard rolled silt Light weight
FU-J T - bars (ISLT)
Ractrr0ular hollow
cloflooE (RHS) Indian Standard rolled silt Medium weight
Hollov, mctoffi (lubEsJ
T - bars (ISMT)
-=
nc
squar. olrcUrrbso
l<oo.'l
l(-l
Indian Standard rolled silt T - bars from
H - section (ISHT)
. Designation: ISNT 125 at274Nlm
)s
ACE'
nneineeringAcad€my
Materials and

d) Rolled Steel Angle sections: . Uncertainf about material strength


. Indian standard equal angles, Indian (Unfavorable deviation of material
standard unequal angles and Indian strength from its characteristic value)
standard bulb angles . Uncertainty about structural dimcnsions
. Designated by abbreviation ISA along (variation member sizes) due to
with lengths of both legs and thickness. fabrication tolerances.
. Indian equal angles are designated as ISA . Uncertainty in the calculation of strensth
or ISEA (ex. ISEA 100 x 100 x l0 mm), of the member.
Indian standard unequal angles are
10. Working Stress Method (WSM):
designated as ISA (Ex ISA 125 x 15 x I0
mm) and Indian standard bulb ansles are
. IS 800:2007 insists for limit state design
and permits working stress method wheie
designated as ISBA.
ever limit state design is not conveniently
7. Analysis and Design: adopted.
.Analysis refers to the determination of the . In this method stress and strain relation is
axial forces, shear forces, bending considered linear till the yield stress. The
moments, torsional moments etc acting on member will be designed for different
different members of a structure due to the load combinations and designed for
applied loads and load combinations working loads.
. Design involves the selection of shape and
. To take care uncertainties in the design,
nel size of the member and connection details permissible stress is kept as a fraction of
of various members (beam to beam, beam yield stress.
nel to column, column to foundation etc) to Stresses due to DL+LL
resist all forces and moments determined < Permissible stress
in the analysis safely and economically Stresses due to DL+WL
8. Design requirements of steel structure: < Permissible stress
. To fit for their purpose (Should sustain all Stresses due to DL+LL+WL
anticipate loads expected on it and Should < 1.33 x Permissible stress
withstand all deformations durins and I. Working stresses:
after construction)
. The stresses used in practical design are
termed as working stress or safe working
. Should be safe
stresses. These should never exceed the
. Should be economical and durable permissible stresses as specified by the
)
9. Uncertainties in design: code.
'
The uncertainties affecting the safety of a
II. Permissible stresses or Allowable stresses:
)
structure are due to . Permissible stresses are some fraction of
ight yield stress of the material. It mav be
. Uncertainty about loading (unfavorable
) defined as ratio of yield stress to facttr of
n
deviation of the load from its safety (FOS) (Ratio of yield stress to
characteristic value,Inaccurate assessment
{r) of the load, improper working stress itself known as factor of
assessment of load
effect etc)
safety) (safety norm in workins stress
method of design).

HvdenbadlDelhilBhopallpmelBhubaneswarlBengarm
lLucknowrpar,alchennairvii"r*ar@
ACE 4 Steel Strucnres
fogloeeriogAcademY

. IS 800:1984 specifies the permissible I of limit states:


Limit State of Strength Limit State of
stress in its various sections
ServiceabilitY
a) Permissible average shear stress = 0'40 & Strength . Deflection

b) Permissible maximum shear stress:0'45 & (yield, buckling)

f; Stability against . Vibration


c) Permissible axial tensile stress:0'60
overturning and swaY
d) Permissible bending tensile stress:0'66 &
Fracture due to fatigue . Fatigue checks
e) Permissible bending compressive stress :0'66fv
Plastic collaPse (including
f) Permissible bearing stress:0.75 & repairable damage
Brittle Fracture
g) Permissible combined bearing and bending due to fatigue)
stress:0.90 &
. Corrosion
III. Minimum thickness of structural steel
. Fire
(ServiceabilitY criteria) :

The minimum thickness of rolled steel


II. Characteristic Strength or Resistance:
section should not be less than The characteristic resistance or strength of a
material (such as steel) is defined as that
a) When steel section is directly exposed to
value of resistanQe below which not more
weather (DEW) and accessible for than a 5 percentage of test results may be
painting and cleaning (ACP) is 6mm
expected to fall.
b) When steel section is directly exposed to III. Characteristic Load:
weather (DEIV) and not accessible for The characteristic load is that value of the
painting and cleaning (not ACP) is 8mm load, which has an accepted probability (95
%) of not being exceeded during the life span
11. Limit State Design (LSD): of the structure.
. The acceptable limit for the safety and IV. The safety format used in Limit State Codes
serviceability requirement before failure is based on probable maximum load and
occurs is called limit state. probable minimum strengths, so that a
ionsistent level ofsafety is achieved'
. Limit states are basically two categories,
strength and serviceabilitY.
V. The design reqttirements are expressed as

follows:
. It is basically statistical method and used
for determination of load and material Design Action (Sd) < Design Strength (Ro)
properties with small probability of 56: Design value of internal forces and
rt.itot. reaching limit state of strength moments caused by the design Loads, F7
and serviceabilitY.
F ,: T7 Characteristic Loads.

Hyderabad lDelhi lBhopal lPune lBhubmeswarlB


5 Materials and Specifications

yf : a load factor which is determined on determined on a 'probabilistic basis'


(uncertainties of element behavior and
Probabilistic basis
possible strength reduction due to
u material factor, which is also manufacturing tolerances and
T
_: imperfections in the material)

VI. Partial safety factors [Load factors (Tr)]:

Limit State of Strength Limit state of Serviceabilitv


Combination
DL LL wL/
EI,
DL LL WLiEL
DL+LL 1.5 1.5 1.0 1.0

DL+LL+WL/EL t.2 1.2 1.2 1.0 0.8 0.8


DL+14r17P4," l.s (0.9)- 1.5 1.0 1.0

This value is to be considered when stability against overturning or stress reversal is critical
a
at Abbreviations: DL: Dead Load, LL: Imposed Load (Live Loads),
fe
WL: Wind Load, EL: Earthquake Load.
be

VII. Partial safety factors [Strength or resistance factors (y",)l:


he Sl. No. Delinition Partial Safety Factor
95 I Resistance, governed by yielding y 1.10
ran mo
2 Resistance of member to buckling y 1.10
a
les J Resistance, gclverned by ultimate stress y t.25
md
a 4 Resistance of connectiofrT
*r Shop Fabrications Field Fabrications

(i) Bolts-Friction Type, \ r.25 t.2s


*,
(ii) Bolts-Bearing TVpe,T r.25 1.25
_u
(iii) Rivets, y,, t.2s t.25
1.25 1.50
(iv) Welds, y
Fa
ACE 6
Steel Structures
nndueeriqgAcademv

correct? Steel
04. Which one of the following is
loads
structures are ideally suitable for impact
because theY have high
(a) Toughness value (b) Elastic modulus
01.In a situation where torsion is dominant'
which one of the following is the desirable itil..tgn stress (d) Plastic modulus

section? factor of
(b) Channel section 05. As per elastic theory of design the
(a) Angle section
(d) Box tYPe section safetY in the ratio of
(c) I-section
(a) Working stress to Yield stress
02.Ideal steel member is to axial (b) Yield stress to working stress
comPressive forces is (c) Ultimate strength to yield.stress
(a) Angle member i0 Uttlmate load to load at Yield
(b) T-section
(c) Channel member 06. The minimum thickness of main steel
(d) Circular hallow section members as per IS800:1984 is directly
for
for carbon exposed to weather, and not accessible
03. Consider the following statements painting is mm
steels with varying carbon content'
As the
carbon content increases
1. Ultimate strength of steel decreases -
2. The elongation before failure increases
3. DuctilitY of metal decreases
4. Ultimate strength increases
Of these, the correct statements are
(a)3and4 !!) 1*9:
ci t. Z and: (d) land2

adalvizag lTinpati lGmuul KukapallvG{vd)


Hvdembad lDelhi lBhopallPme lBhub
Riveted & Bolted Connections
el
ds

4. Classification of rivets:
Simple Riveted & Bolted Connections
(a) Based on method of heating:
o Hot driven Rivets: Rivets are heated red
of l. Rivet and Riveting: hot before driving
Rivet consists of head and shank and are o Cold driven rivets: Driven at atmospheric
made of mild steel or high tensile steel. The
conditions without heating.
rivet head is generally round and called a o Cold driven rivet has more strength
button or snap head. The size of rivet is the
compared to hot driven rivet.
diameter of the shank.

(b) Based on method of placing:


3el 2. Holes are drilled or punched in the connected
o Work shop rivets
tlv members to be riveted. The size of rivet holes
are 1.5mm larger than the size of rivet up to
o Field driven rivets
[or
25mm, for rivets of
diameter more than
o Work shop driven rivets have more
25mm, the diameter of hole should be 2mm strength than freld rivets
larger than the diameter of rivets (c) Based on method of driving force:
o Nominal diameter of rivet ( 0 ) is the
o Power driven rivets

diameter of shank (Diameter of rivet


o Hand driven rivets
before riveting process)
. Power driven rivets have approximately
20o/o more strength than hand
driven rivet.
According to the Unwin's formula the
nominal rivet diameter is assumed as
5. Important rivet heads:
$: 6.04 1t"tr.n $ and t are in 'mm')
^[
O
: l.9l .,/t (*fr.n $ and t are in ,cm,)
o Gross diameter or effective size of rivet
(d)
: diameter of rivet hole (Diameter of
rivet after riveting process)
d:0+1.5mm if 0<25mm . Snap head or Round head rivets mostlv

d:0+2.0mm if d>25mm
used forconnecting structural steel
member. Flat counter sunk or round
counter sunk head rivets are provided to
3 Grip of the rivet or bolt is the distance have flush or flat surface.
between the undersides of the two rivet heads
(i.e., shank length inside the connection) and 6. Bolt and Bolting:
grip of rivet should not be more than g$. o { bolt may defined as metal pin with head
at one end a shank threaded at other end to
receive a nut. Steel washers are usually
HvdenbadlDelhiiBhopallPunelBhubanewarlBpngalurulLucknowf Pdnlalchenmilvijayamdalvizas
lTirpati lqurt4l lfubtr1p,6ed)
,fiti- .AqEa-rff
trIjfnOrcgint.{cadery :8: SteelStnrctures

provided under bolt as well as under tt


e (b) B-ased on the type of force experienced by
[
pressure on
t, -L^ LL^ ^l^*-inc hfAccrrfe
nut to distribute the clamping
^unJio"prevent on I the bolts:
tne uuns;
I
the bolted membe, the I ':htT::T::*l:
:ff.#ffi"Jilil;.ffi"ild;;;;g
-^e^r -i^^^o
on the connected Pieces'
-o
I
connections and

' Combined shear and tension


I' I'*i:l
Connections

. Bolts can be
ttolts U4u ur\ for making
ue used end
| ' -_1ff-]--.l
.J----I-E-.
connection in tension and compresslon
hold
I
members. They can also be used to I A,t,,pConnoction

column bases in position. I tt euconnectbw I

+crr&' 9\^tor'

Sryprt

T-P
BOtT W
IETIGTH

faJ"
oJn-lor t,
'
rrfl J a,,-1"
l.
'

nfle(ltDn TauionpluShlur

HEXAGOt|AL SQUARE
HEAD EOIT HEAD BOLI

(c) Based on force transfer mechanism by


BOtT ASSEilBtY

bolts:
1. Classification of bolted connection: o Bearing type (bolts bear against the holes
to
based
The bolted connections are classified transfer the force)
into three
on g.o-.tty and loading conditions o Frictio the
types namelY plates ated

(a) Based on the type of resultant force bY the

transferred:
o in
Concentric connections (force transfer
tension and ComPression member)
o
Eccentric connections reaction (in
transferring brackets) or moment resisting
connections beam (in
column to
connections in frames)'

(r) ft)
Corcnfilc Cott$flnc lloncil Cuneobnt

Hyderabad I Delhi I BhoPal I Pune


*8i*ffi** 9 Riveted and Bolted Connections

8. Types of Bolts: The HSFG bolts are available in sizes


They are several types of bolts used to from 16mm to 36mm and are Cesignated
connect structural members. Sor,ne of them as Ml6, M20,M24 and M30.
are listed below: These bolts tightened (bV torque
wrenches) until they have very high
(a) Black Bolts or unfinished bolts : tensile stresses, so that connected parts are
. Black bolts are referred to as ordinary, clamped tightly together between the bolt
rough or common bolts. They are least head and nut, this permits load to be
expensive bolts and are made of low transferred primarily by friction not by
carbon steels (mild steel) with square shear.
or hexagonal head. The diameter of These bolts are most suitable for bridges
the hole is about 1.0 to 2.0 mm larger where the stress reversal may occur or
than the bolt diameter for ease in slippage is undesirable also for seismic
fitting. They are designated as (M) dx/ loading and for fatigue load.
'd' - shank diameter of bolt and'l' - High strength bolts have replaced rivets
length of the bolt and black bolts are being used in
They arc primarily used in
light glgg$fql, .high raised building, bridges
'g structures under static loads such as
small trusses, purlins, bracings. They
:lc
9. Types of Shear connections:
are also used as temporary fasteners
by They are two types of bolted joints subjected
during erection where HSFG bolts or
to axial force (the loads are assumed to pass
welding are used as permanent
through the C.G of the group of bolts)
fasteners.
o These bolts are not recommended for (a) Lap connection:
the
connections subjected to impact load.. The two members to be connected are
ted
vibration and fatigue 6ru,t- ,*lry#, overlapped and connected together
. For bolt of a property class 4.6 such a joint is called lap joint.
represents the ultirnate tensile strength The load in a lap joint has
is 400 N/mm' and yield strength is 0.6 eccentricity, as the center of gravity
times 400 which is240 N/mm'. of load in one member and center of
o Ordinary bolted joints, the force gravity of load in second member are
transfer through interlocking and not in a same line, therefore a couple
bearing of bolts and joint is called formed which causes undesirable
bearing type joint. bending.
(b) High Strength Friction Grip (HSFG)
Bolts:
o High strength friction grip bolts are
made from bars of medium carbon Don.ble boltttl lap joirtt
, rrr' ' 'heat
treated iteel ltrigh tensile steel).
,(' The bolt property class 10.95 and
12.95 are commonly used in steel Siqele Ril'eted lap conaedioa Double riv-eted lap eoraettion
connections. Lap Connecfi,oas

Hydenbad,Delhi;Bhopalihne:BhubmeswlBengalmlLucknow PatnalChema'VijayamdalVizg ]Tirupati lGunul Kukapally(Hyd)


rftu, ACE : 10: Steel Structures
SDifngitledhgecademY
:
(b) Butt connection: . For double cover butt joint, each cover
a Two members to be connected Placed plate thickness is 5/8 x thickness of main
plate.
end to end. Additional cover plate/plates
are provided on either one or both sides,
called cover Plates are Placed and b. Tacking rivets of bolts (Stitch rivets or
connected to main Plate. bolts): Used to make two sections act
If cover plate is provided on one side it together and in compr
prevent buckling when
is called single cover butt joint.
contact. These bolts are
If cover plate is provided on both sides
calculated stress or loads
of main plate, it is called a double cover
butt joint.
Double cover butt joint, eccentricity of a
11. Specifications for Riveted and Bolted
connections:
force doesn't exist and hence bending is
eliminated, where as it exists in the case
o Diameter of rivet hole (d) and diameter of
bolt hole (d")
of lap joint.
The shear capacity of bolt in double
o Pitch (p)
cover butt joint is double that of a bolt
o Gauge (g)
in a lap joint. o End distance (e)
r Edge distance (e)

H +- Garuge

#r
,&'H
td Ed Firch
DutN t am, e r bnfr joint (D u$le tultu iI)

Enddistame

centers of two
- ' Pitch (p): Distance between
(a)
consecutive rivets or bolts measured parallel
Siagle ivet€d sirgle eover Double rivetd single Gover
to the direction of the force in the member
bctt counectiou
hett soaaection
lying on the same rivet or bolt line' It is also
defined as C/C distance of rivets measured
along the length of the member or connection
in case of wide plates. When rivets or bolts
Sirgle rivetd dodle €oroer Doubte riveted dosble cover are placed staggered the pitch will be refened
bltt coenectbe hutt couuectioa to as staggered Pitch
Bntt Counectbos Minimum pitch is ensured for the following
reasons
. For single cover butt joint, cover plate o To prevent bearing failure of connected
thickness required is 1.25 times the main member between two rivets or bolts
plate.

viiavawada vize Tirupati Gunnr I KukapallvHvd)


Hyderabad I Delhi I Bhopal I Pune I Bhubaneswu I
Bengalm I Ilcknow I Patna I Chenrni I I I I
q#ffi^*"* : ll : Riveted and Bolted Connections,

To permit efficient installation of rivets or (c) Rivet or Bolt line_(or gauge line):
bolt, prevent overlapping of steel washers Line along
which rivets or bolts are placed.
etc
Minimum pitch:2.5 x nominal diameter (d) End distance: It is the distance between
the
of rivet or shank diameter of rivet (>2.50) center of rivet hole or bolt hole to the nearest
or 2.5 x shank diameter of the bolt (>2.5d)
edge of main member of cover plate edge
measured parallel to the direction of a load
J Maximum pitch is ensured for the member.
in
0 following reasons
To reduce the length of the connection (e) Edge distance: It is the distance between
)
the
and gusset plate i.e to have a compact center of rivet hole or bolt hole to the nearest
joint. edge of main member of cover plate edge
To have uniform stresses in the rivet of measured parallel to the direction of a load
in
bolt member
Maximum pitch : l6t or 200mm o
Minimum end/edge distance
whichever is less for tension members :
1.5 x Gross diameter in case of rolled,
Maximum pitch : l2t or 200mm machine flame cut edges (1.5 d)--
whichever is less for compression member MinifriiniTndiedge distance
(Where t : thickness of thinner connected : 1",7.L*Qfgg;diameter of rivet for
member) sneai.?-otlffil*name cut edges( I . 7 d)
32t or 300mm whichever is less for o Maximum end/edge distance
tacking or stitch rivets or bolts (when : 37m,m_ * 4t (IS800:I984)
plates are not exposed to weather) (o M^aximum edge distance to nearest edge
l6t or 200mm whichever is less for
L of bolt hole to an edge of un stiffened part
tacking or stitch rivets (when plates are should not exceed l2te *h"..
exposed to weather)
In case of two angles, channels or tee t =./l
r;\l & 40 mmr4t where t is
section maximum pitch of tacking rivets -
v(t',
(In which tacking or stitch rivets uL to U. thickness of thinner outside where t is
provided along length to connect each of thickness of thinner outside plate (for
them) corrosive Environments) (IS g00:200D
I Not exceeding 600mm for compression

I
members
Not exceeding l000mm for tension
ofsimplerffi
members.
Working Stress Method

(g): Distance between adjacent rivet 1.2. Failure of riveted connections:


.Gauge
lines or CIC distance between two A riveted connection is subjected
consecutive rivets or bolts measured at right load, it may have chance io fail
angles to the direction of the force. It is also possible ways
defined as clcdistance between two (a) She4f failure of rivets: The failure mav
consecutive rivets measured along the width occur when the shear stress in the rivet
of member. exceeds the working shear stress in the rivet.
: 12: Steel Stucnrres

Shear stresses are generated because P]*S (e) Tearing failure of plates: The tensile stress
to applied forces. in the plate at net cross section may exceeds
;ligdue the working tensile stress. Tearing failure may
occur when rivets are stronger than the plates
under tensile force.

Shear faflnue of rivet


ffi**
Teauiag failure of plate
(b) Bearing failure of rivets: The rivet crushed
half circumference. The plate may be -Emng
in bearing and heaviest str6phte may crush (f) Edge cracking or splitting failure of plates:
the rivet shank. This failure may not occur in Rivets may have been placed at lesser edge
practice except plate may strong in bearing. distance than required causing plates to be
split out or shear.

Beariog {aihre of rivet


Selittir€ failare of plate

(c) Bearing failure of plates: The plate may be


crushed when the ing qfe_s'q..q-r the plate 13. Assumptions in design of riveted
exceeds working bearing stress connection:
. A load is assumed to be uniformlY
distributed among all the rivets.
\
")) {, Shear stress is assumed to be uniformly
distributed over its gross area of rivet.
Bearhg failnre of lrlate
a Ben{ing stress in rivets is neglected.

(d) Shear failure of plates: The internal pressure


a Friction between plates is neglected.
of over driven (shank length is more,than a Rivet hole is assumed to be completely
gnp) rivets are placed at a lesser edge filled by the rivet.
distance than specified cause this failure. This Bearing stress distribution is uniform and
failure can be cipg$:d by providing proper contact area is d x t where 'd' is gross
egggglsta+ce between center of rivet hole and diameter of rivet and't'is thickness of the
end*F
of the plate. plate
The tensile stress is uniformly distributed
on the portion of plate between rivets

Sheer feihre of Plate


lres
l3 Riveted and Bolted Connections
tress 14. Permissible stresses in work shop rivets:
;eeds
(b) Strength of rivet in bearing (p5):
Strength of one rivet in bearing
may Axial tension
Type of rivet Shear Bearing pu
lates (MPa) (MPa)
=dxtxoor
(MPa)
Power driven Strength of riveted joint in bearing
100 100 300
Hand driven 80
Po =dxtxoorxn
80 2s0
opr: Permissible bearing stress in rivet.
. For field rivets, the permissible stresses t - thickness of thinner plate in lap joint
rivets are reduced bv l0%
: thickness of main thinner plate or sum of
. The permissible stresses in rivets and bolts cover plates thickness, Whichever is
may be increased by 25o/o, if the effect of minimum in case of a double cover butt
wind or earth quake load is considered. joint.

15. Strength ofriveted connection or . Rivet value or strength of rivet (R" or p)


Joint
(P. or P1): Minimum of strength of rivet in shear (ps)
and strength of rivet in bearing (p5).
(a) Strength of rivet in shear (p,) Ru or P.: Smaller of p. and pr,
Strength of one rivet in single shear (p,)

nd2
P. Determine the rivet value of 20mm diameter
rivets connecting lOmm thick plates by using lap
'eted
Strength of one rivet in double shear (p,) connection. The permissible stresses for riveis
in
shear and bearing 80 Mpa and 250 Mpa
rmly respectively and for permissible bearing stress
P. = 2. ,*r.r,, plate is 270MPa.
in

rmly
16 : Permissible shear stress in rivet.
Sol:
Strength of riveted joint in single shear
Gross diameter of rivet, d:20 * 1.5: 2l.5mm
nd.2
ty1: 80 MPa and opr:250 Mpa
P, 1- X.t_
"
etely 4vl Strength of one rivet in single shear (p,)

Strength of riveted joint in double shear n nd' nx2l.52


L and
'4u'4
ross nd2
P,
rf the
4
= 29.04 x 103 N = 29.04 kN

Where permissible or allowable shear Strength of one rivet in bearing (p6)

stress in rivet Pu = d" 1 >< opr = 21.5x l0x 250


d: gross or effective diameter of rivet =53.75x103N=53.75kN
n: number of rivets in a lap joint Rivet value is smaller of p, and pu:29.04 kN

Hydembad I
Delhi I Bhopal I
pue Bhubme
I
ACE : 14: Steel Structures {
nndneemg4".4".Y
plate in tension (P)'
I
. Number of rivets (n) required to suPPort ' - Strength of
(c)
Tearing strength of Plate (P):
a

working axial load in case of a concentric


connection P, = An"t X 6ot = (B -nxd)xtxou,

-PRu Where,
An.t : Net effective sectional area of plate
B: width of the Plate
n: Number of rivets at the section
t: Thickness of thinner Plate
A 8mm thick angle section is joined to a 12mm
oat : Permissible axial tensile stress
load in
,ttl.i. gutr.t platel The angle is supporting a
of gO fN. Find out the number of 18mm
diameter plate: 0.6xfy
! : yield strength of a Plate
rivet d: 18mm it is :
. Strength of joint (Pr or P)
Gross diameter of rivet d: 18
+ 1'5 : 19'5mm
minimum strength of riveted connection
(P')'
based on strenglh of rivets in shear
For power driven rivets strength of rivets in bearing (Pb) and
Permissible shear stress in rivet t6:
100 Mpa
strength of Plate in tension (P)
Permissible bearing stress in rivet,
oor:300 Mpa
or joint (q):
16. Efficiency of the connection
(P')
Strength of one rivet in single shear
Strength of rivetedconnection or Joint
x 100
P.= XTuf =--4 ^" n= Strength of solid Plate
O
=29.86x103N = 29'86kN : fs-x1oo
P,o
(Pu)
Strength of one rivet in bearing
Po = dxtxoor = l9'5x8x300
*ou'
N = 46'8kN 17. Strength of solid plate P,n: Ae
= 46.8x 103
is Where,
The strength of one rivet or Rivet value A*: Gross sectional area of the plate: B't
i*u of siength of one rivet in shear (P') and
rtt."gtft oi one rivet in bearing
(Pu):29'86 kN
Number of rivets used to
Working axial load A singlerive ,is
n= -- Riu"ftul"' connect two ivets used
;;; p"*.t driven 16mm rr a Pitch of
in shear
_P_ 85 =2.82= 3 ;brrim. The permissible stresses for rivets
Ru 29.86 u"J L"uti"g tOO\Afu and 300 MPa respectively
unJ fot p"ermissible tensile stress in
plate is

: ,: : :: :-; :'^--^:trn:." rrtrlYizag lTtrupad icunffl KukaDanjG+va)


Hyrtembad I Delhi I BhoPaI I
uffi-"m^*"' :15: Riveted and Bolted Connections

150 MPa. Find out the safe load per pitch length Prp: Ae Xoat: Pxt xout: 50 x 12 x 150
and efficiency of the joint.
l.6mm d rivets
:90 x 103 N:90 kN
rz** ! Strength of riveted Jointperpitch
* Tl = x 100
Strength of solid plateperpitch

Sol: Nominal diameter of rivet d: 16 mm


= 4r 100 =
48'1
x 100 = 53.4%
Gross diameter of rivet d: 16 + 1.5 : l7.5mm P,o

For power driven rivets


Permissible shear stress in rivet tur: 100 MPa
Permissible bearing stress in rivet, opr:300 MPa
Permissible tensile stress in plate, out:150 MPa A tie member has to transmit a pull of 320 kN.
Design a butt joint to connect it with 200 ISF12
Pitchp:50mm
is flat. Also find the efficiency of the joint.
Strength of one rivet in double shear per pitch The permissible stresses for rivets in shear and
ton
length (P.) bearing 100MPa and 300 MPa respectively.
lr),
* A2 nxlT .52 Assume yield stress of steel250 MPa.
rnd
P =2x'u- xr. ^ """'"
Trt=/'
=2x xl00
' 4 4 ' Sol: Assume nominal diameter of rivet
=48.10x103N =48.10kN (ns
\-v 4:
O.ot Jt
''
:6.01x JtQ :t9 mm = 20mm
Strength of one rivet in bearing (P6)
Minimum pitctr of riiet p,,'in : 2.5$
Pr =dx tx oor =17.5x12x300
:2.5 x 20:50mm
= 63.0x l03N = 63.0kN
Assume thickness of cover plate
Tensile strength of plate per pitch (P1)
t"o:5 ^2:7.5^m= Smm
8
P, =Ao",;1 out =(p-nxd)xtxou, Gross diameter of rivet
=(50-1x17.5)x12x150 d:20 + 1.5 :21.5mm
= 58.5 x 103 N = 58.5 kN Permissible shear stress in rivet

Strength ofriveted joint is minimum of tur: 100 MPa


strength of rivets in shear (P,), strength of Permissible bearing stress in rivet,
:d to rivets in bearing (P6) and strength of plate in opr:300 MPa
used tension (Pt)
Permissible tensile stress in flat,
:h of Strength of riveted joint P.; :48.10 kN
ihear
on1:0.6x x250:150 MPa
f;= 0.6
ively Axial Pull P:320 kN
Strength of solid plate per pitch length (P,o)
;e is Strength of one rivet in single shear (P,)

Hydembad I Delhi I Bhopa.l ! Pme j Bhubmew Bengalm


I I
Lucknow i Patrra I Chemai I Vijalamda lvizg Tirupati I Gutur
I I KukatUally(l lyd)
L ACE :18: Steel Strucfirres
inogineerinsAcademY

20. Design strength of bearing type bolted In long joints, the distance between the first
connection (Va.): and ttre tast bolt exceeding l5d in the
(a). Design shear strength of bo-lt1(Va,u): direction of load, the nominal shear capacity
Design shear capacity of the bolt (Va't) V,,t, shallbe reduced by the factot,pri'
V, (0.75<prj <1.0)
Vo.o =...9
Y.b
Prj=1'075 k
Where,
Vnrb: nominal shear capacity of a bolt I is the length of joints (Distance between

f-.
.*t.-. end bolts measured parallel to
V",o =if (nnAno +n.A,5) direction of connection)
{J prris reduction factor for long grip lengths'
f...
V,"* =-]L(nnAnu + n.Aro)
r/3 yn.o

Where,
f*:ultimate tensile strength of the bolt
: its shank
nn number of shear planes with threads
intercePting the shear Plane B,j =
-!9
3d+ (.,
n, : number of shear planes without threads
intercePting the shear Plane /*: griP length (/, should not greater than 8d)
Fnur: reduction factor
for packing plates
As6: rlorrlinal plain shank area of the bolt
: n&14 When packing thickness is more than 6mm
thick the shank of the bolts is subjected to
A16: flet tensile area at threads, (area
bending which affects the nominal shear
corresponding to root diameter at the
capacity of the bolt
thread = 0.78 A.t) For.* =1'0-0'0125t't,
Tm
: Partial safety factor for bearing type tptg: thickness of thicker packing plate in mm
bolt
:1.25 (for workshop and site bolting) (b). Design bearing strength of bolts and Plate
(Vapn):
For bolts in single shear, either nn or ns is one
A bolt bearing on any plate subjected to a
For bolts in doub[q;shear the sum of nn and n'
is two. factored shear forcg
The nominal shear capacity of bolt for lgqg

\1
.f joint is lesser and modified as
f
Vaou
V'pb
=;ry
Imb
whereY.6 =l'25
v",o =*(flnAnu +n.A,o)Bo, F,rPoo,
{J Vnpb:nominal bearing strength of a bolt'
f. /
=2.5x dx tf" xko
vo.u =*- (n Anu + n,A,o )F,ip,rpou, Vo,b
{ Jy-r "

reduction factor for long joints,


rdalvizs l Tirupati l Gunff l KukaFallv(tlvd)
tures n affio*"-, l9
Iryt d and Bolted Connections
Where,
e first (d). Design Tensile strength of plate
(Tapl,
n the
rcity ku =smaller of
$, '(3d" --
3d" [3-o.rrJ ,
fu andl.0
),
))
4: diameter of the bolt hole

e & p : end and pitch distances of the fastener


respectively along bearing direction
rtween
lel to
fur : ultimate tensile stress of the bolt prerEs wrtn Borrs Hous rw Telstox
fu : smaller of the ultimate tensile stress The tensile strength of plate by net
of section
lhs, the bolt and the ultimate tensile stress rupture is given by
of
the plate
_ 0.9 rA, f,
'if the d: nominal diameter of the bolt in mm
Too "
T^t
rminal
t- summation of the thicknesses of the Netarea =A" =(b-n
;reater "
do )x t forchainboltine
connected plates experiencing bearing
Lng on
stress in the same lirection. The tensile strength of plate by gross
section
yielding is given by
\/ooo ---=-V"ou 2.5xdxtf,, xk.
v, mb Tmu
T
,
* __y*_
-A*"f'
(c). Design Tensile Strength of
bolts (Tar):
21. Design bolt strength or Design bolt
value
The nominal tensile capacity of bolt in (Vou):
tension
It is least value of design strength of bolt in
is given by
(
shear ign strength- of bolt in
bearing design stringth of bolt in
T"o =0.9d0 Ao6 Sfp A.o b tension (T66) (if exists).
y,no

f*:ultimate tensile stress of the bolt


2.5
&u: yield stress of the bolt
The bolt safe in tension if the factored tension
force
T
Too =-t wherey.o =1.25 andy,oo =1.10
Tmu

rtub ^ f"'A"
Too =9.9 ^nu a r" su
Sol: For bolts of grade 4.6, :400 N/mm2
{,6
Tmb Y,no For Fe410 grade plates, f,, :410 N/mm2
&
Tnu:1.25
In lap connection, the bolts will be under
single shear and bearing, hence desisn
ACE :20: Steel Strucnues
nngineeringAcaaemy

strength of a bolt is least value of design


strength of bolt in shear (Va,u) and design Design or Factored axial load
strength of bolt in bearing (Vapu) Design strength of onebolt OD

Design strength of bolt in shear (Va't)


f.
%,u =#L(nnAno + n.A,o)
r/J Y-t
Two flats (Fe 410 grade steel), each 300mm x 16
0.78l(ro)' +o; =28.97x ldN mm are to be connected using 20mm diameter
=-i00-11
.lz"t.zs' x 1x
4' bolts of grade 4.6 to form a lap connected' The
connection is supposed to transfer a service load
=28.97kN
of 375 kN. Calculate number of bolts required for
connection with minimum pitch and end distance
Design strength of bolt in bearing (Va*) for bolts. Assume thread of the bolt doesn't
intercept the shear Plane.
r,'opD 2.5kbdtf,
Imb Sol: For bolts of grade 4.6, fu6:400 N/mm2

-.-- rr^-.^r e -
,Ku=Sfflallefol- 3o 0.55; For Fe410 grade Plates,
3xlg
=
(r:410 N/mm2 & yrnu : I'25
Diameter of a bolt d : 20mm
( :--o.rrj =f -1L-o zsl= o +q Diameter of a bolt hole do:22mm
[3d" J \r,.ts )
Minimum pitch of the bolt p :2.5x20:50 mm
fuo
Minimum end distance e =1 '5 x22:33 mm
4oo
= - o.g7 &r.o
fu 410
n : Design load (P) / Design strength of one
ku = 0'49 bolt (V66)
Design load (P) : L5x375:562.5 kN
2.5x0.49 x16x 10x 400
V.,
'opD
t.25 Design strength of bolt (Vau) is least of Vo'u
or V6o6
= 62.72x 10'N
= 62'72V,N Design strength of bolt in shear (Vo't)
Design strength of bolt (Vat) is least of V6'6 f.
%* =*(nnAnu +n,A,o)
or V6n5 {J T_u

Design strength of bolt is28.97lN (Ans) + t x t' 1(zo)' ) = 58.04 x to3 N


= -44-1o
Jl"t.zs' 4'
22. Number of bolts (n):
If the connection is subjected to concentric
design axial load (P), the number of bolts (n) Design strength of bolt in bearing (Vopu)
required to suPPort design load is
ACE'
EngineeringAcademy 2r Riveted and Bolted Connections

2.5kb dtf"
Vooo =
rmD
Compute the design strength and efficiency of a
e 33 bearing type connection as shown in the figurc.
3d"=3x22 =0.50t
k^=smallerof
" The full threaded bolts are grade of 4.6 and,
20 mm in diameter and plates are Fe410 grade.
x16
reter
The
(;-,,') (#-o 2s)= = o 5o7;

load
400 'l
l for 35mm

ance 4t0
= 0.97 &1.0 rl120mm
I

]sn't -E
| 35mm
+

35mm 65 mm 35 mm l5mm 65 mm 35mm


25x0.50x20x 16x 400 12 mm thick main plate
=128x10'N=128kN
1.25 :400 N/mm2
Sol: For bolts of grade 4.6, f,u
Design strength of one bolt (Var) is 58.04 kN
For Fe 410 grade plates,
Number of bolts n: f":410 N/mm'& y-o
I.25 :
P/Vau:562.5158.04
Diameter of a bolt d = 20 mm
Diameter of a bolt hole do:22 mm
= l0 (Ans)
rmm Minimum pitch of the bolt p : 65 mm
23. Design strength of a connection (V6.): Minimum'end distance e:35 mm
mm
In double cover butt connection, the bolts
I one
It is a minimum design strength of a bolted will be under double shear and bearing &
connection is based on strength of bolt in plates in tension hence design strength of a
shear (V6,6), in bearing (Vopu), in tension (T66) connection is least value of design strength
(if exists) and minimum design strength of of bolt in shear (Va,r,), design strength of bolt
'Voru connected member against gross section in bearing (Vapu) and design strength of a
' yielding or net section rupture (T6o). plate in tension (T6o)

Assuming thread of the bolt intercept the


24. Efficiency of the joint or percentage shear plane (nr:0)
strength of joint (q):
Efficiency of the bolted joint (q) also called Design strength of bolt in shear (Varu)
percentage strength of the joint is the ratio of f
design strength of joint to the design strength %.u =f!L (rtnAnu + n.A,6 )
103 N of main member expressed as percentage. AJ T.u
Efficiency of a connection (4)
400 n
= -----,- _-(4x2, o.tt + o)
l'l = x 100 43x1.25 4'1zo),
= 362.1 g x I 03N = 362.1 gkN
.ftF-#"-."* 222: Steel Structures

Efficiency of a connection
Design strength of bolt in bearing (Vat)
2.5kb dtf"b (rl): Va"t100/T'p
Vuno =
Ymt :362.19 x 100/518.18

:--o.rr) :69.89
= [-q--o.zs)= o zr;
(

[ra" -)\3x22 )
Yo

fut 4oo Clats Room Practice Questions


fu -
- 0.97 &1.0
410
kt = 0'53
j!-=0.53; 01. Which one of the following bolt is desirable
ku =smallerogJ- = when structural bolted connection is subjected
to dynamic or imPact loads?
(a) Unfinished bolt
\/tdpb - 2.5x0.53x20x 12x400 (b) Close tolerance bolts
1.25 (c) High strength friction grip bolt
(d) Ordinary bolt
= 407.04x 103N = 407.04kN
02. When the distance between centers of two
Design tensile strength of main plate (T6n)
adjacent bolts connecting the members
0.9A-f 0.91(190 - 2 x 22) x t}lx 410 suLjected to either compression or tension
'
do=--- r.25 exceeds the maximum Pitch, then the
additional bolts which are not ected to the
T-t
= 517.1904 kN
calculated stresses are known as
(a) Packing bolts (b) Long-grip bolts
Design strength of connection (VaJ bolts (d) Auxiliary bolts
: ici Tacking
362.19 kN
03. The maximum longitudinal pitch of bolted
joint subjected to tensile force' where 't' is
Efficiency of a connection (n)
Design strength of connection (VJ thickness of plate and 'D' is diameter of bolt
q=@idprate(T,n) x 100
is
(a)32D (b) 16 D
Design strength of solid plate (T'o) (c) 32t (d) 16 t
0.9A"f, 0.9 (190 x 12) x 410
^sp ^v 1.25
04. Which one of the following failure can be
eliminated by using minimum end distance
and pitch disiance as per IS 800 provisions?
= ozl.lsxlo3N = 673.05kN
(a) Tearing failure of Plate
Design strength of solid plate on yielding (b) Block shear failure of Plate
(c) Shearing failure of bolt
An f" (190x l2)x250 (d) Tearing failure of bolt
^sP v =
1.10

= 518.18x103N = 518.181cN
adalvizg l Tirupari l Guurl Kuka4allv(Hvd)
L .,.-

ffi*ffis"-, :23: Riveted and Bolted Conne<:tions

(05)In triple bolted double cover butt joint, the


[08/A member of a truss consists of two anglcs
- design strength of connection per pitch length - ISA 100 x 100 x 6 placed back to back. lt
in shearing the bolt is ,'fj times the carries an ultimate load of 150 kN ancl is
design strength of one bolt per pitch in single connected to a gusset plate 8 mm thick placccl
shear, where n is equal to in between two connected legs. Assumc
(a)2 (b)3 @)a (d) 6 Fe410 grade of steel. End distance and pitch
distance to bolt holes are respectively are 30
06. Mild steel flat subjected to a design axial mm & 40 mm. Partial factor for bearing type
force of240 kN is joined to a gusset plate by bolt is 1.25. The number o 6
double cover butt connection using bolts. If bolts of grade 4.6 required
rable the design forces required per each bolt (i) to (a)2 (b)3 (c) a (d) s
ected shear the bolt, (ii) to crush the bolt and (iii) to
tear the bolt are 40 kN, 50 kN and 30 kN 09. The desien tensile strength of M16 bolt of
respectively, then the number of bolts grade 4.6 is KN
required is (a) 50.2a kN (b) 30.14 kN
(a)4 (b) 6 (c) 8 (d) 10 (c) s1.13 kN (d) 43.86 kN

07. The tension and shear force (both in kN) in 10. The design strength values of a bolted
two each bolt of the joint, as shown below, connection are 180,000 N against tearing,
nbers respectively are 160,000 N against shearing and l, 50,000 N
nsion against bearing. If the design strength of the
the solid plate is 2, 40,000 N; the efficiency of
to the the lap connection is %
(a) ss.0% (b) 62.s%
ls (c) 66.6% (d) 7s.0%
S

rolted for
't' is (a) 30.33 and 20.00 (b) 30.33 and 25.00 01. (c) 02. (c) 03. (d) 04. (b) 0s. (d)
f bolt (c) 33.33 and 20.00 (d) 33.33 and 25.00
06. (b) 07. (d) 08. (b) 0e. (d) 10. (b)

an be
;tance
ns?
Welded Connections
. In the case of tension members' tt:
1. Introduction to welding or welding
absence of holes imProves
the s[en$h
process: andefficiencY of the sectron'
.' the process of joining two
"ofis
cost compared
W.taing
bY creating a
il;;;il.. 6ss fabrication
pi.""t me strong handling
;;;;i';;thods due to oPerations of fewer
them bY like
metallurgical rnd between both' It is ;;;^;"J etmination of consequentlv
(fusion) or presswe or iliil;;.rnctting etc' and
heating 'f'o'n other forms of to economY
distinguishtJ less labor leading-uit
such as riveting tigltt and water tight
mechanical connections' ffi;i;; ofiers
by friction
or bolting, which are formed filft;d hence is ideal for oil storage
or mechanical interlocking' tanks, shiPs etc'
Welded structures are more and
rigid
lllleld machine or Power
riveted
source and controls
compared to structures with
Elestrode Holder bolted connections'
formed by
A *fV continuous structure is
;.-"*"t*s of fusing the members
tosether.
strong or
CJ*rallV welded joints are as-
;;;;;;'tnun ttt" tuttonmetal' therebY
joints'
pi*iig no restriction the is
Stress concentration effect -also
For steel structures metal of
elecffic
less in a welded connection'
IJ"tiao"urv are
welding is generallY used
the i"*. the disadvantages of welding
for
In all -od"* are welding Processes' for the tft"i i "frequires skilled man power
*""'is shielded (flux coating)
arc is shteldeo welding as well as insPection'
may
following reasons Ail;, non-destructive evaluation-
. fo p-i.ct the molten or liquid metal i,r,uu.',o be carried out
to detect defects in
from air, either with gas vapgr' ffit-P.fects like internal air pockets
for
'io .orrtiol the melting of electrode ;;;-= incomPlete Penetration' cracks'
energy
more effective use of are porositY).
. To add alloYing elements 'W"iaittg in the field may be difficult due
of welded Welded
2. Advantages and Disadvantages to the location or environment'
under
connections:
more
over ss)
Welding offers many advantages on are
. Latge residual stre -
develoPed in welded connectrons'

is minimum.

."n-'*el
:25: Welded Connections

3. Assumptions in the analysis of welded


connections:
The following assumptions are made in the
analysis of welded connections
he . The welds connecting the various parts
,glh are homogeneous, isotropic elements
. Only stresses due to external loads are
red considered. Effects of residual stresses,
vef stress concentrations of welds are
ike neglected.
rtly . The parts connected by the weld are rigid
c) Classification based on type of joint:
and their deformations are neglected
ght (i) Butt or groove weld (ii) Lap or fillet weld
age 4. Types of weld joints: (iii) Tee weld (iv) Corner weld

,gid Welds can be classified as follows . It was seen that welded connections are
and continuous and more rigid when
a) Classification based on type of weld:
compared to bolted connections. It was
(i) Butt or Groove weld
tbv also pointed out that fillet welds and butt
)ers (ii) Fillet or lap weld welds constitute respectively 80% and
15% of all welds in the construction
Ior industry; the balance 5% is made up by
'eby plug, slot and spot welds.

also 5. Design of Butt (Groove) weld:


on. . Butt welds are used to connect structural
i afe members that are aligned in the same
for plane. Edge preparation becomes
necessary for plates over lOmm
may thickness for manual welding.
ts in . A square groove weld used to connect
:kets plates up to 8mm thickness.
rcks, . Butt weld is usually designed for direct
n .A_A
Section A-A tension and compression and shear also
: due in some special cases.
rlded
rnder a) Reinforcement:
more . It is very difficult to maintain flush
;) b) Classification based on position: surface during welding with parent
n are (i) Flat weld (ii) Horizontal weld member
(iii) Vertical weld (iv) Overhead weld . Reinforcement is exha weld metal
(0.75mm - 3mm) which makes throat
dimension at least I0%o greater than the
thickness of the weld material. The reason
HydenbadlDelhilBhopallPunelBhubaneswarlBangalmlLucknowlPanralChennailViiayamdalvizs lTrnrpod lGunltt,l Xfttrdpfi$)
*ftt ACE
iEDirnginrx.ugecademv ;26 z
Steel Strucnrres
w-

for providing reinforcement is to increase State Design ConcePts)


fl,imit
the efficiencY of the connection of butt weld
. Reinforcement makes the butt weld
stronger for static loads. However the
(D
,.irrfot..-ent effect is neglected in design
strength calculations

&1
: smaller of yield stress of the weld (t*)
and parent metal (!) in MPa

L*: Effective length of weld in mm


b)- Types of butt weld:
S[,rate butt weld, Single V butt weld, Double
tr: Effective throat thickness of weld in mm
V butt weld, Single Bevel butt weld, Double : 5/8 x
Thickness of thinner connected
bevel butt weld, Single U butt weld' Double member in case of single butt weld (t)
U butt weld, Single J butt weld and Double J : Thickness of thinner connected
member
butt weld etc. in case of fully penetrated butt weld (t)

fi::5q::3 t-\h EgFJd['fr ^Il**: Partial safety factor against weld


'
ffi* SngleV Daftley S:irrg/eFervl
strength

'":{*F:E n:{:l:
{l
E*F*$=
U
fi*ffi J
ym*:1 .25 fot work shoP welding and
siile Aeret
n $rngie Donila Srrrg/e

ym*:1.50 for site (freld) welding


ffid:f
Du6leJ
Ilp.g€BtttMitt

(ii) Design shear strength of butt weld


c) Size ofbutt weld (t"): Oo.'n)
. The size of butt weld is specified by throat The design strength of butt weld in shear
dimension and also called effective throat is governed bY Yield

vu*=@
T**
fwt : smaller of shear stress of the weld
welds). (&*/,6 ) and parent metal (&/J: ) in MPa
:
t. Thickness of thinner member in case of
Double V, Double U and Double bevel butt
joints (i.e for fully penetrated butt welds) thick Plates are joined in the
workshop by (i) a single 'U' butt weld (ii) a
d) Effective sectional area (A.): double butt weld. The effective length of weld is
The effective area of butt weld is the product 300mm. Determine the design strength of welded
t
a

of effective throat thickness and the effective joint as per limit state design of 15800:2007' The
length of the butt weld.
HydembadiDelhiIBhopallPunelBhubme'*"1S"t*lutlI-"kto*lP*"1Ch"MlVti"*d"l% l@
Academy :27 : Welded Connectiorrs
yield and ultimate tensile strength of weld and
steel are 250 MPa and 410 Mpa respectively.

Sol: For workshop welding y^*:1.25


(Tu*)
eld in f,:410 N/mm2 & f,*:410 N/mm2
red by fr:250 N/mm2 & &-:250 N/mm2
TXpicol Jillet t' el d rcn n e'ri on s
(i)For single 'U' butt weld (partially penetrated
butt weld, as per specification) I. Specifications of fillet weld:
Effective throat thickness t.: 5tl8 :5x l6lg a) Size of fillet weld(s): It is minimum les
(&*) length of cross section of fillet weld (it is
:1Omm
Effective length of weld L*:300mm distance from root to the toe of the fillet
Strength of butt weld (T6*) weld)
mmm
b) Minimum size of fillet weld (sn';n): The
Lnected 300x l0x 250
size of a
fillet weld should not be less than
td (t) t.25 3 mm or more than the thickness of the
rember =600x103N =600kN thinner part joined
rld (t)
(ii) For single 'U' butt weld (For full penetrated Thickness of thicker part Minimum
butt weld, as per specification)
Up to and size of fillet
Effective throat thickness t.: t:l6mm Over (mm)
Effective length of weld t*: including (mm) welds (mm)
300mm
Strength of butt weld (Ta*) a
t0 J

l0 20 5
300x l6x 250
20 32
I2s 6

=960x103 =960kN 8 (First run)


JL 50
&10
Design of Fillet welds (or) Lap welds:

e weld The fillet weld is done for members which Maximum size of fillet weld (sn'u*):
LMPa overlap each other or used to connect : Fillet weld is provided to square edges,
structural members that are aligned in the two
the weld size should be at least 1.5 mm
different planes, for such welded joints shear
less than the edge thickness (i.e Thickness
in the stress are critical. Usually convex or concave
of thinner member - I .5mm)
(ii) a
veld is For the rounded toe of a rolled section. thc
velded weld size should not exceed 3/4 thickness
7. The of the section at the toe (i.e : 3l4tt
thickness of rolled section at toe)
j Vfayawada I Vizzg I Tirupati Ourrtrr
I I X,,l,,rr,,,llvtt tvO
lll.. AcE
:SitnF;ectintAcadeq'
:28: Steel Strucnues
rt-

'let welds on
ftltet welds
sqillrc edge o[
0n squarc plole'
o[Plole, fillet welds oil rlttttd loe oltolletl scclion

d) Effective throat thickness (t):


:' It is perpendicular distance from right
angle corner of fillet weld to the SSetr sress
along welil tltoat
hyPotenuse.

o Minimum throat thickness of fillet weld


not less than 3mm (15800:2007
Specification onlY)
. tt : K x Size of the weld (K: constant (ii) In order to control it the length of the
faces
depends on angle between fusion '--'
longitudinal (side) fillet weld should
not
. Fillet welds are normally used for U" i..t than the width of the
plate' The
form
connecting parts whose fusion faces tn.u.n stress distribution increases as
the
angles between 60o and 120'' width of theglate increases'

e) Effective length of fillet weld (L'"): cnd *eld


. It is actual length shown on the Flrt bar to guccet Pbte
drawing

L* : gr,.rall length of weld(L) - 2x (iii) For this reason the perpendicular distance
*^^'u*..n
size offillet weld (2s) the longitudinal frllet weld is
Minimum effective length not less limited to 16 times the thickness of
than four times size of weld (4s) or ,hi*.t plate joined' If the plate is wider
may
40mm whichever is higher' itran ttris limit slot or plug welds
(D The stress distribution in the welded
joint int odoced, which tend to improve the

."*pft* and non-uniform' The actual


it distribution of stress in the plate'
variation of shear stress distribution
in the
(iv) It is assumed that the strength
of
fto* point A to B depends on- the
*.iJ '^''longitodinal and transverse fillet weld as
tength of weld and also the ratio of
the
;ti,hplates being joined' Thus it is
of shoivn in figure below is same' Actually
seen tha-t there is serious stress ihe strength of transverse fillet weld is
plate'
concentration at the edges of the
rcs
!lFaginceringecaaemv
ACE
;29: Welded Connections

about 30%o more than the longitudinal


fillet weld, because transverse fillet weld
is stressed more uniformly for uniformly Two plates of 8mm and l2mm thickness are to be
for full length whereas transverse fillet joined using fillet weld. Suggest suitable size of
weld is stressed non-uniformly due to fillet weld and length of end retums.
varying deformations along the weld
Sol: Minimum size of fillet weld s,o;n= 5mm
I Maximum size of fillet weld
l
s'o*: 8-1.5 :6.5mm (for square edges)
Adopt size of fillet weld s 6mm :
End refurns :2s:2x6 :I2mm

g) Minimum overlap:
The overlap of plates to be welded in lap
Lotrgituditral fillet,$eld Tralsverse filletrveld
joint should not less than four times
the thickness of thinner member (4t) or 40rnm.
not f) End Return: whichever is more.(Note: 15800:2007
the a The fillet weld terminating at the end or Specification)
the side of the member should be returned
around the corner when ever practicable
for a distance not less than twice the weld Oveilap 6 at or 40 mm
size as shown in fisure

Note: Overlap in lap connection should not be


less than 5times thickness of thinner
member (50 [IS800: I 984 Specification]

End returns are made twice the size of the II. Intermittent lillet weld:
weld to relive the high stress
o Intermittent fillet weld: when length of
concentration at the ends fillet weld required transmitting a force
End returns must be provided for welded less than the continuous fillet weld (where
joints, which are subjected to eccentricity, t is thickness of thinner plate)
stress reversals or impact loads. This
particularly important on tension end of
parts carrying bending loads.
Most designers neglect the end returns in
the effective length calculations of the StaggcredlnbmifteBtfill€twetd ChaininbrmitlentlllLtvucld

fillet weld. End returns must be provided {ntermihnl ffletweld


for the welded joints that are subjected to o Clear spacing between intermediate fillot
eccentricity, stress reversals or impact weld should not more than
loads.
v
t
t
ACE Steel Strucurres i
:30:
Eoeioc€triogA.ad€ny
strength
weld (t), the reduction in weld
.l2t or 200mm whichever is less for as Per the long joint'
comPression member . Th; design caPacitY of weld (f*d)
.16t or 200mm whichever is less for
reduced bY a factor
tension member
Bn*=',-9t
150tt "'
lnterrnittent
frllet weld t
/i : Length of joint (or) length of side weld in
' the direction offorce transfer
elcments t: Effective throat thickness of the
weld
+)
Staggered ,c
intermi$ent
frlletweld between
Determine size, length and overlap
i) tf"r.t of the fillet welded lap connection to
"rr"*Lii , factored load of 250 kN as per limit
a"tig" of 15800:2007 as shown in the figure'
,^tut.
:- Design strength of fillet weld
(Pa*):
and ultimate tensile strength
of weld
III. il;;t.d
ift. iesign shear of fillet weld f*6: f*'/]'n* unJ *i..f are 250 MPa and
410 MPa respectively'
and width
f*o : Nominal shear strength
of fillet weld Assume workshop welding Y**:1 '25
: of the plate is 100mm'
ful Ji
The design strength of a fillet
weld (based
on the throat area)
t*
(t<.s)f"

in mm
L*: Effective length of fillet weld
tt: (K s)Effective throat thickness in mm
s: Size of weld in mm
Smaller of ultimate strength
of weld
fo:
and Parent metal in MPa
:1 '25 for Factored load P:250 kN
y** = Partial safety factor [Y*Y
:1'50 for 3mm
shoP welding and Tmw Minimum size of fillet weld S'o;n:
site (field) weldingl Maximum size of the fillet weld
smo:10- 1.5:8.5mm (for square
edges)

Joint (p: AdoPt size of fillet weld s:6mm


^' - Reduction factor for long
IV.
Specification only)
Effective throat thickness tt: K
Q'{ote: 15800:2007
s:0'7x6
= 4.2ttwr
If the maximum length I of the side
its length
wetds transferring shear along
size of the
exceeds 1Jn tirnes the throat
ACE
ures nngiocedngAcadcmy
Welded Connections

)ngth By Equating design strength of fillet weld to


factored load
(f*a)
Po*=p=L-F'th -p
{3y.* =12.49xI06
L.. x4.2x410
# 43 xl.25 =250x103 +L* =314.31mm 7. Design of Plug (or) Slot Wetds:
,ld in
Plug and slot welds are used most often to tie
Effective length of weld L*=314.3Imm two parts together and in particular to reduce
Overlap between plates (providing weld on the unsupported dimensions of cover plates in
all three sides as shown in figure) compression. There may also be used for
shear transmission.
Overlap = (3 14.3 1- I 00)/2:107. I 5mm
Plug and slot welds are used along with fillet
Meen (Minimum over lap as per 15800:2007 is 4t weld, when sufficient welding length is not
nto or 40mm whichever is higher) available along the edges of the members.
limit A slot is cut in one of overlapping members
gure. and having welding metal is filled in slot if
weld the slot is small and completely filled with
vely. A circular plate, 100mm in diameter is welded to weld metal. It is known as plug weld
vidth another plate by means of 6mm size fillet weld as
shown in figure. Calculate ultimate twisting The following specifications are for design of
moment capacity that can be resisted by fillet plug on slot weld as per IS 816 1969.
-
welded connection as per limit state design of
15800:2007. Use steel of grade Fe4l0 and
workshop welding.

nA-A
,7
Slotweld

(a) The width and diameter of slot should not


be less than three times the thickness of
part which slot is formed or 25 mm which
ever is gteater.
(b) Corner at enclosed ends should be
Sol: For workshop welding T^w: I.25 rounded with a radius not less than 1.5
f,,:410 N/mm2 & f,* :410N/mm2 times the thickness of upper plate or 12
Size of fillet weld s:6 mm mm which ever is greater.
Effective throat thickness tt =K s: 0.7 x 6 (c) The distance between the edges of the
:4.2mm plates and slot between edges of adjacent
J Length of fillet weld L*: n d,:3.142 x 100 slot should not be less than twice the
m thickness of the upper plate.
=314.2mm
Ultimate twisting moment capacity To
ACE 232 Steel Shuctures
Engine€riogecaaemy z

8. Weld svmbols: Design strength of fillet weld: P6*

Eatr
llFa elFdd
;lrlD $Pot nry Flr{d
Klc
: P4*_rt P6*,
v n€ft| J Trtt Itwrl trlti vrK
1.55 :217.6 kN
5qsrtl U
lro{t tlord toa{ hilt
ratl$ Pa*:106.05+1 1
mt frca fatr cdtc*

N tl V Y v Y r ,t # a7l C Service load carrying capacity :211.6 I 1.5


Elrofolf,{5rrrt ct slElot Itrllor, of l
:145 kN
Fkl{tbm8 0iercd fttDi

Cffi,il
Working Stress Method Concepts
f;oqt{{e

9. Strength of fillet weld or Lap weld (P.):

Strength of fillet or Lap weld Pr: L* .tt. rur


Determine service load permitted on fillet welded
connection as per limit
design of state L*:Effective length of fillet weld
IS800:2007as shown in the figure. Use steel of tt: Effective throat thickness : K s
grade Fe410 and site welding.
s: Size of fillet weld
tur= Permissible shear stress in the fillet weld

10. Strength of Butt or Groove weld (T.):


Strength of butt weld, T": L*.1. .otr
Ir: length of the butt weld
t.: effective throat thickness
otr= Permissible stress in tension or
Sol: For site welding T-*: 1.50 compression of the parent metal.
fu: 410 N/mm2 & f,*:4lo N/mm2 The permissible stresses in the joining thembers
Size of fillet weld s: 8 mm of mild steel by weld
Effective throat thickness tt:K s: 0.7 x 8
. Permissible shear stress in
fillet weld
:5.6mm tur: 108 MPa
. Pednissible stress in tension or compression
Design strength of side fillet weld (l0mm of the parent metal otr: 150 MPa
fillet weld)
. Permissible bending stress in tension or
size
compression of the parent metal obc: obt :
p. L*F ){, 120x5.6x410 =106.05x10N
165 MPa.
^dw s = =
-
l^
{JY.* lExl.5 . Permissible stresses in the welds are reduced
by 20% when the welding is done in the freld.
Design strength of plug weld (30mm plug weld) . When effects of wind or earthquake loads are
considered, the permissible stresses are
4to increased by
D ]ooy'x =111.55x103N
^d* p
fxl.s
ACE
Engine€ringAcademy : 33: Wekled Connct:tions

Sol: Allowable tensile stress in plate


Find the safe load that can be transmitted by the o4: 150 Mpa
fillet welileAioiii shown in figure below. The Allowable tensile stress in weld
size of weld is 6mm and allowable stress in the rui: 108 MPa
weld is 108 MPa Let'x' be the overlap between plate
The welded joint will be designed on rhr:
basis of strength of smaller width plate

120
T Load allowed on the smaller width plate (p9
mm
Arr oat: 100 x l0 x 150: 150 x 103 N
.-."P5
T Size of fillet weld s:6mm
Effective throat thickness
tt:Kxs:0.7x 6:4.2mm
Sol: Permissible stress in weld 16:108 Mpa
Effective length of fillet weld
Effective length of fillet weld L- :2
xl50+120
L*:(2xx+100)
:42A mm
Shear strength offillet weld (p,)
Size of f,rllet weld s:6mm P, :L*X tt X Tvf: L* x 4.2x 108
Effective throat thickness tt: K xs
Equating safe load allowed on the fillet
: 0.7x 6:4.2 mm welded joint (P) : Shear strength of fillet
Equating safe load allowed on the fillet welded weld (P.)
:
joint (P) Shear strength of fillet weld (p.) 150x103:L*X 4.2xI0g
:
P:Pr: L* x tt X Tvf 420 x 4.2 x I08 :(2xx+100)x4.2xI0g
: 190.51 x l03N: 190.51 kN
Overlap x: lI5.36mm = l20mm

A 100mm x 10mm plate is to be welded to


Two plates 160 mm x 10mm and 160 mmx
another plate 150 mm x l0mm by frllet weld on
l2mm are to be butt welded. Calculate the
three sides. The size of weld is 6 mm. find out the
strength of butt welded joint in tension. If (a) sing
necessary overlap of the plate for full strength of
V butt weld, (b) a double V butt weld is used to
)n or the joint. Take allowable tensile stress in plate
connect them. Take permissible stress in weld as
equal to 150MPa and allowable stress in weld as
)t- 150 MPa
108 MPa
duced Sol: Allowable tensile stress in weld
T
field. T oor: 150 MPa
ds are
i are
l'j loo
trlm
Effective length of butt weld L*: 160 mm
1
I (a) For single V butt weld
l- L
I
Effective throat thickness te: 5tl8
:5x10/8:6.25 mrrr
:34: Steel SMrctures

: L,'t" (P) : Shear strength of fillet weld


Tensile strength of butt weld T, 'etr
:160 x 6.25 x 150 : 150 x 103 N :150 kN (P.) 150 x 103: L*x 4'2 x 108

o) For Double V butt weld L*:330'68 mm


Effective throat thickness t": t: 10 mm
The distribution of the weld should be such
:
Tensile strength of butt weld T, L.' t" 'o6 that the c.g. of the weld coincides with the
:160 x 10 x 150 :240 x 103 N =240 kN line action of the load
Let Lwr and Lwz be the length on heel and
toe of an angle resPectivelY

L'o1* L,u2 : L* : 330'68mm---------(1)


Taking moments of loads and weld strengths
about c.g of an angle

Pslx 19.7 -P52

\, x19.7-R, x45.3=0
stress in the weld as 108 MPa

P., t9.7 L*z rY' I

P:! l-Lw: +{
L*r : 2.299L*z-----(2)
From Equation(l) and Equation(2)

L*z:100.24 mm
rsA65 X65 X 10
L*r230.44 mm

Sol:
Let 's' and L* be the size of fillet weld and
effective length of fillet weld respectively'
Axial tensile load P : 150lcN: 150 103 N
x

Minimum size of filet weld s,o6: 5mm


Maximum size of frllet weld
s^u*:3 xl0l4:7'5mm
Adopt of size of fillet weld s: 6mm
Effective throat thickness
tt:
K x5:Q.Jx 6:4.2mm
Equating tif. load allowed on the fillet
welded joint
lvizag i Tirupati l Gunu'l
r"ada Kuka$allv(Hvd)
:35 : Weldecl Conncclions

Class Room Practice Questions

01. An angle member is connected to a gusset


plate as a strut in a roof truss, to achieve full
such
strength of weld ensured, if its maximum size
h the
for an angle member is limited to
(a) 12s kN (b) 130 kN
(a) l.5mm less than thickness of angle leg
I and (c) 140 kN (d) 135 kN
(b) 75% of the thickness of angle leg
(c) Thickness of the angle leg
(d) 1.5mm higher than thickness of the angle 06. Two plates of dimensions 150 mm x 16 mm
leg
and 150 mm x 12 mm at their welding edges
are joined by butt welding as shown in the
02.The weakest plane in a fillet weld is figure. What is the maximum tension that
(a) A side parallel to the force
this single V-butt weld joint can transmit?
(b) A side normal to the force The permissible tensile stress in the plates is
(c) The one along the throat 150 MPa.
(d) The one normal to the throat Topr t in 15
{oft hdh tufres)
03. A fillet weld of size s as shown in figure. The
effective throat thickness of fillet weld along
throat is (where o is angle between weld
faces)
(a) 0.70 s (a) 168.7slcN (b) 270.00 kN
(c) 218.00 kN (d) 13s0.00 kN
(b) 0.6s s
NOnnl'
(c) 0.60 s 07. A circular shaft of diameter I20 mm is
(d) 0.s5
welded to a rigid plate by a fillet weld of
s
size 6 mm. If a torque of 8 kN-m is applied to
the shaft, what is the max m stress in the
04. The effective length of a fillet weld of overall weld (to the nearest unitf tvt
length 'L' and size of weld is 's' is given by (a) 80 N/mm2 (b) 85 N/mm2
(u)r- rE (b)L-2s (c) 90 N/mm2 (d) 95 N/mm2

(c)L - st Ji (d) L 08. Two plates are connected by fillet welds of


size 10 mm and subjected to tension, as
;lls. A fillet-welded joint is shown in the figure. shown in the sketch. The thickness of eaclr
i
f the weld is 8 mm.
r 1r^^ -- -rr : o
plate is 12 mm. The yield stress and thc
in the weld is 110 ultimate tensile stress of steel are 250 Ml,a
What is the safe force (to the nearest and 410 MPa, respectively. The welding is
magnitude) to which the weld can be
done in the workshop (T-* : I.25). As por tlrc
subjected?
Limit State Method of IS g00.2(x)7. 't'lrt.
HvdenbadlDelhilBhopallPuneiBhubmeswulBengalmllmkrowlPaunlChemiiViiayawadalVizg
iTirupari lCurrrur'l Kllrtprllvflv,l)
|
Steel Stnrctures
:36 :
l*#Acadcmv
(a) l,2and 3 @) 1 an{ ?,otlY
minimum length (rounded off to the
nearest
t.iz'*a 3 onlY (d) 1 and 3 onlY
higbti,*tttlele of 5mm) of each weld to
2701d'{ is
transmrt a cteslgn rorce P equal to
12. Arangle
ISA 90 x 90 x 8 (Cvv: Cu = 25 mm)
used as
(a) 100 mm
tie member in a truss is welded to 12 mm
(b) 105 mm iiti.t got.tt plate using maximum n]rmissilf1
weld size on top and bottom edges of
an t
(c) mm
length. The yield stress.and the ulti
110
il;g
(d) l l5mm i.ntii. ,n-.tt of steel and welds are 250 MP
and 410 MPa, respectively' The
tie member i
subjected to service load of 200
td\f'
tto**J
*riding is done at site (T.*: 1'5) Tg
09. Two plates of 10mm and 20mm
are joined factor [ ,r:t.5.The ratio of toP weld
work and botiom weld length of an
with doubls '{J' hutt weld at fabricated
. ", rft"oft^*: I.2 e yield stress and the 1S
".$
S'- .)P
-\P Xlntti*ui. t."tit.
-\*),,r"-ji" tensile of steel and welds welds are
13. For field welds, th
s

\t{Z 250 MPa and 410 re^sryctirre! The


weld are reduced bY
effective length of weld is 300mm'
As per
t# ii-ia ,,ur. aesign of IS 800:2007 what can be
J.rign load allowed eld
is kN 01. (b) 02. (c) 03. (a) 04' (b) 0s' (d)
(b)
10. When length of side fillet weld
is 300 times 06. (a) 07. (b) Os.O) 0e' (600)10'
the design
the effective throat thickness' Then 11. (d) 12. (2.6) 13. (20)
per 15800:007
shear capacity of fillet weld as
1S

(a) IncreasedbY 20% (b) Decreased bY 20%


(c) Increase bY 40% (d) Decrease bY 40%

f lltgt welds
2. Irlrei carry the tou0"anot"8[pil*
WEI\rD vqrJ -7\rr[-Otr '
6"..4 on the - tegsiles'"-slrength
of fusion
characteristics material'
3. For effective transmission of load by fillet
weld, the fusion faces shall subtend an
angle between 60" and 120o

Hyderabad I Delhi I BhoPall


"*bfra-h-*,-]5

Eccentric Connections
sed as
2mm
E ccentric B olted. cotutection
o A third type connection which resists end
issible moments as well as permits relative
angle rotation between the beam and column.
timate Introduction: The moment rotation relationship for
Connections become complex when they have
) different types of connection are shown in
rber is
to transmit axial and shear forces in addition figure
t. The to bending moments or twisting moments,
between structural members oriented in
d load klenlly rigid
different directions. Rigid connection
length
Concentric load: t
I ,Semi rigid csnnection
A load is said to be concentric ioad, when its
line of action passes through centre of gravity p
ises of bolt group or rivet group or weld group o Flexible connection
et *
Eccentric load: --?F*'e Ideally flo,
-rdnellv flexible
H
A load is called eccentric load when its line of
action does not pass through centre ofgravity ltototicrn (0)+
, (d) of bolt or rivet or weld group. Because of
eccentricity additional moment is induced in f,. Analysis of Bolt Groups:
t. (b) the joint.
In
general, any group of bolts resisting a
moment can be classified into either of two
Beam to Column Connections: cases depending on whether the moment is
Beam to column connections can be classified acting in the shear plane (Bracket type
as simple, semi-rigid and rigid depending on connection-I) or in a plane perpendicular
the amount of moment transfer taking place (Bracket type connection-Il) to it.
between the beam to the column.
. Simple connections are assumed to
transfer only shear at some nominal
eccentricity. Therefore such connections
can be used only in non-sway frames
where the lateral loads are resisted by
some alternative affangement such as
bracings or shear walls.
. Rigid connections transfer significant
moments to the columns and are assumed
to undergo negligible deformations. Rigid Brecket Type Connection-I
connections are necessary in sway frames
for stability and also contribute in
resisting lateral loads.

HvderabadlDelhilBhopallPuelBhubanemlBengalmlLucknowlPamalChemailVijayalvadalvizag
1t-pail1C*uot f,*24g$fffW.
ACE :38: Steel Stucnues
ynngioceringAcadenY

r of bolt from C.G of


- Radial distance :>*
bolt group; >f +4?
The resultant of Fu and F* act on the bolt is F
and 0 is angle between Fu and F., the resul
force Fn

. Critical bolt is that which is subjected


the maximum resultant force. Critical bol
which is farthest from C.G of the bol
Bracket QPe Coanection-Itr
group and nearest to the applied load li
is most critical.
. For safety of bolt group
6. Bracket Type Botted Connection-I @lastic (Design requirement) Fn-* S
Analysis): strength of the bolt (Vaul
(a) Load or moment is lying in the plane of . For design of connection aPProxi
Bolt grouP: p number of bolts in each vertical I

nt=

n' - No of bolts or rivets required per


vertical line
m - No of bolts or rivet lines in vertical
p - Pitch of the bolt or rivet
Bracket plate

l-section cslumn Vor - design strength of one bolt

The eccentric load P may be replaced by


concentric load and in plane moment Working Stress Method ConcePts
I

(Twisting Moment) (P x e) acting on the joint.


I

Bolts are subjected to direct concenffic


I

For safety of rivet group connection,


factored load (P) and a twisting moment criteria FRnu* < Rivet value (P'orR')
(M=Pxe) For design of joint approximate number
Direct vertical shear foroe in each bolt due to
Direct axial load Fu: P/n Rivets in each vertical line n'=
(n - Number of bolts in connection)
Force in each bolt due to moment (M:P.e) n'-No of rivets required per each vertical li
F-:P er l2f m - No of bolt or rivet lines in vertical
n Number of bolts/rivets in a
- p - Pitch of the bolt or rivet
connection R, - Rivet Value or Strength of one rivet
P - Factored load
e - Eccentricitv of a load
Hydenbad I Delhi I Bhopal j Prne I Bhubmesw I Bengalm I Lucknow I Parna I Chemi i
vijapwada I
vizg Tirupati
I I
Grmur I
ACE
tures Engio€cringAcad€my : 39: Eccenfic Connections

of
FR:113.1 kN

A bracket connection is made with five Ml6 bolts


lisFn
and supports a factored load of P:100kN with an
ultant
eccentricity of 200 nun as shown in figure below.
The maximum force taken up by any bolt is

(a) 20.0 kN 10
;ted to l-
cos0:'" =0.6
al bolt (b) 70.7 kN
?@ 50
e bolt 4{ rl 40 mm
ld line (c) 113.1 kN {: $r

(d) 100.0 kN
4U.
3* *e 30 mm
rection 30 30
design (b) Bracket Type Bolted Connection-Il(Elastic
A[t$tl6 qffirr,'vwl
Analysis)
ximate Sol:An eccentric load may be replaced by set of . Load or moment is not lying in the
al li one direct concentric load (P) and in plane plane of Bolt group
moment (M)
P

P: 10 kN
T
M: P.e:100(200) :2xI0a kN-mm
i
]
i

n
Bolt No's l, 2, 3 & 4 are corner bolts and i

having maximum radial distance, bolt no I

I & 2 are nearer to applied load line and T'Beacket


J-
subjected to maximum resultant force I*sectiot c{)lunrn

Force in each bolt due to direct concentric The eccentric load P may be replaced by
load (F,) concentric load and moment (P x e) acting on
the joint.
spts
Fu:P =]!q=2okN Bolts are subjected to Direct concentric
n5 factored load (P) and a bending moment
(M:Pxe)
Force in critical bolt due to moment (F-) Bolts are subjected to direct shear along with
Tension due to moment
mber F-: P't:t - 2x10o x50 = l00kN
lI Er' 4x 50' + 0 Vertical shear force in each bolt due to
Direct concentric load Vu :Pln
Ru Maximum resultant force in critical bolt (n - Number of bolts in connection)
ical li | &2is
_ Tensile force in any is bolt due to moment
rl Fn.*:JFi +Fu2 +2 F"F..cos 0
.r! M'Yi
-br
r-
=-

Lvet = 4202 + 1002 + 2x20x 100 x 0.6 Zv?


:113.1 kN

I
:40 Steel Stnrctues
ecademv :

Moment of resistance provided by bolt in Tuf,"ul otf ,"ol


* < 1.40
tension M' (Assuming axis of rotation at h/7 Tuf otf

from bottom of the bracket) t.,01: Permissible shear stress in the rivet

otr: Permissible tensile stress in the

rivet.

For design of joint approximate number of


Tensile force in the extreme critical bolt

t -M'Yn Rivets in each vertical line n'= 0.8


'o-Iyi
n Number of bolts in connection n'-No of rivets required per each vertical line
-
m - No of bolt or rivet lines in vertical
e - EccentricitY of load p - Pitch of the bolt or rivet
h - Height of bracket R" - Rivet Value or Strength of one rivet
. For safety of bolted connection is checked
Yr,Y2,..., Yn - Distance of each
bolts in
in combined shear and tension by using
tension from the axis of rotation interaction equation of IS800 :2007
/ \2 /- \2
Working Stress Method ConcePts l5l*l 5l <ro
Iu"./ [T"/
Vu: Factored shear force on bolt,
Bracket Type Riveted Connection-Il
Vau: Design shear caPacitY
[When no initial tension Rivet (i. e cold
driven rivets)l: T6 : Factored Tensile force on bolt,

Calculated or actual shear stress in rivet Vau: Design Tension caPacitY


T vf :cal :Y6ln @)2A
. For design of connection approximate no
(Where Vu: Vertical shear force in each
of bolts or rivets in each vertical line
due to direct concentric load bolt: P/n)
Calculated or Actual tensile stress in
rivet o tf ,cal: f,"ln (d)21+
(Where T6: Tensile force in the extrerne
m - No of bolt or rivet lines in vertical
p - Pitch of bolt or rivet
Design strength of bolt or Rivet
rivet due to Moment M' is ! =S3l Vou -
Value (P'orR")
Lv,
o For safety ofriveted connection is'checked in
combined shear and tension condition to be
satisfied as per IS800:1984

HydenbadlDelhilBhopallPuelBhubmesrulBangalmlLncknowiPatnlChermailVijavawadalVizg lTirupati lGuntrl


ACE
:4L: Eccentric Connections

Weld is subjected to Direct concentric load


(P) and a twisting moment (M: P x e)
The tension and shear force (both in kN) in each bolt Direct vertical shear stress in the weld
of the connection, as shown below, respectively are q1 :P/(d+2b) t'
d - Depth of the bracket
tt - Effective throat thickness of fillet weld
I t
I I -Ks
a I Maximum shear stress in weld at due to
twisting moment (P.e)
260 kN gz:Pern'u*/In
Io- Polar moment of inertia - I** * Iyy
Sol: Resolving force 260 kN in to horizontal and P - Factored load
vertical components; Both force components e - Eccentricity of the load
are passing through C.G of connection r - Radial distance of weld from C.G of
Cos 0 :l2ll3; sin 0: 5/13; weld group
n: number of bolts present in bolt group: 6 . The resultant stress of qr and q2 act in the
weld is gn and 0 is angle between gr and
Horizontal force component produces tensile
Qz, the resultant force
qa
force in each bolt :260 cos0/n
cked
260 qi +ql+2q,q, cos0
rsing - 613 xP = 4okN

Vertical force component produces shear For safety of connection the resultant
force in each bolt : 260 sin shear stress (qr)^* S Design shear
0lo26o ,1= 16'67kN
(r)
strength of weld I f*o = f T** | as Per
613 \ t/J )
Limit state design of 15800:2007 (or)
E ccentric Welded corute ction :
@n)^* 3 rur permissible or allowable
shear stress in weld 108 N/mm' as per
IS800:1984 (Working Stress Method
(a) Bracket Type Connection-I (Fillet principle)
Weld):
Load or moment is lying in the plane of
fillet weld group (Elastic Analysis):
Determine the maximum resultant stress and its
location in the fillet weld ABCDA for the bracket
rtical connection as shown in the figure. The size of
weld is 6mm.
Rivet

Bracket plate
I-seetion colunrn

I
ACE z 42:.
Steel Strucnues
nnsin€cdngAcadeinY
material is
When welding is done half of the
converted to liquid.
Heavily stressed point is one at which
the
Sol:
which is
resultant stress in weld is maximum'
nearest
farthest from C.G of weld group and
i"-ift. applied load line, Hence Point B and
Point C aie heavily stressed points

An eccentric load may be replaced by set


of
in plane
one direct concentric load (P) and
moment (M)
P: 150 kN ,
M: P.e. : 150 x 350 k]'{ - mm
notut MI of fillet
weld
Sf
EccentricitY of a load
:Irrll,

e:200. +=35omm r, :ru =@4#S = 7 5'6rx 1 o6 mma

Radial ai.t-*.. to heavily stress


point B;

r-*:J150' + 1502 --212.1 mm Ip:Ir,* Iyy : 75.61 x 106 + 75.61 x 106

^ 150 : 151 '22 x 106 mma


"oto=L=o-7
Size of fillet weld s :6 150 x 103 x 350 x 212'13
=i3.64N/mm2
\z =
l5l.22xto
Effective thro at thickness - point B
Maximum resultant shear stress at
tt:0.7 x 6:4.2mm
Vertical shear stress due to 150 kN (or) point 'C'

150x103
o': (9n)-u*: e
i;{=G;oo);Gt
:29.76N/mm2 cos 45o :0.707
(M)
Shear stress due to twisting moment
= {2g.7
( * 73.642 + 2x29'7 6 x73'64 x 0'
M
Ot:
t = 96. 83 N/mm2

1502 +1502 = 2t2.l3mm

: ,: : :. ;;^:* ndalvizg lTirupui lcuurl ruutpalvGrvd)


Ilyderabad I
Delhi I BhoPal I
:43: Eccentric Corurections

(b) Bracket Type Connection-Il (Fillet (c) Load or


moment is not lying in the
Weld): plane of Butt weld group:
Load or moment is not lying in the plane of
fillet weld group:

/l cl

Bracket platu
[-section eolnmn
rf*l4r irchel
Direct vertical shear stress in the weld
I*secti$u colrutru
ga: P/ d te
Weld is subjected to Direct concentric load of the bracket
d - Depth
(P) and a twisting moment (M: P x e)
t, -Effective throat thickness of butt weld
Direct vertical shear stress in the weld Bending stress in weld due to moment
(P.e)
Qlcat: Pl2dtt
d - Depth of the bracket tn : P e (dlz) I [(t"d' ll2)]

tt - Effective throat thickness of weld (Ks) e - Eccentricity of the load


Bending stress in weld due to moment (M:P.e) d - Depth of the bracket

The combined stress of f.ur and act on


e2cat:P e (dl2) I 12 (tta3nZ71 Qcar
the butt weld is f"
lmm2
e - Eccentricity of the load
I =Jf3, +3q3",

int B d - Depth of the bracket The combined bending and shear stress in
. The combined or equivalent stress of
the butt weld may be checked by the
Qrcar
and Qzcar act on the fillet weld is f.
interaction formula as per limit state
design of 15800:200'
f-= ,. f..
I
=r/f"i^, +3ql,r <---r--
I rn0
For safety frllet weld group connection the -
As per IS 800:1984, if butt welds are
equivalent shear stress,(f,)-u* 5' Design
(f\ subjected to shear and bending, equivalent
shear capacity of weld I f*a =*y.* | as stress< Permissiblc value o equivalent
\\/J./ stress: 0.9 f"
per limit state design of IS800:2007
(f,)r* ( :
permissible or allowable
rur
shear stress in weld 108 N/mm' as pe.
IS800:1984 (Working Stress Method
principle)

Hydenbad I Delhi I Bhopal I Pune I Bhuburesm I Bengalm j Lucknow I Patn I Cbennai I Vijayawada I
Vizg Tirupati I Gunor I KukapallvHvd)
I
Steel Strucmres
z 442

t
t.0
a)

T*

rdf,

force (both in kN) in


< 1.0 04. The tension and shear
.u.fr Uof, of the joint as shown below,

(c)

<t.4

"-'
02. Locate the critical bolt in the bolted
as shown in the figure below (a) 30.33 and 20'00
,o*..tion
(b) 30.33 and 25'00
(c) 33.33 and 20.00
(a) BoltNo.l
(d) 33.33 and 25'00
(b) BoltNo.3

(c) BoltNo.4 05. Which one of the following


is the most
design of a
important consideration in the
(d) BoltNo.6
bolted connection between structural
members when the centroid
of the bolt group
J*t- coincides with the axis of-load?
"", in
\--l Oit*, shearto its radial
t"l force each bolt is
proportional distance from
in
its centroid rnd the resultant force
each bolt should not exceed its strength
of bolt.
bott due
ft)
\v'' The shear force caused in each
IS 800:2007? a'-....n*titY of the 1o19 is
its radial
proportional to
dlstance
ffi ACE
nngineeriogAcad€my :45: [pcentic Connections
from its cenhoid and direct shear
force in each bolt should be limited to
the half the strength of bolt.
(c) The shear force caused in each bolt due
to eccentricity of the load is
proportional to theradial distance of
the bolt from the cenhoidof bolt
group and maximum resultant force in
any bolt should not exceed the strensth
of bolt
(d) The strear force caused in each bolt due r,rfl)'*fl')'
' \160/
[48/
<t.4
$. to eccentricity of the load as well as
r^ directs shear force caused in each bolt
should not exceed the strength of bolt
individuallv. rur [l'')'*ft)'
\40l \30/
< 1.0

06. A Moment of magnitude g0 kN_m is < 1.0


transmitted to a column flange through a
bracket using four M20 bolts as shown in the
figure. The shear force induced in the bolt A
is
<r.4

08. A Bracket plate connected to a column flange


-l'
transmits a loadof 100 kN as shown in the
C'l
n.-L
! following figure. The maximum total force
for which the bolt should be designed
is kN
00
r most
rofa (a) 250lclt{ (b) 200 kN +-
600

ncfural
group (c) 12s kN (d) 88.4 kN
+a ror
t?
75
ls
}7.Each bolt shown in figure is t)
olt is capable of loiG
resisting design shear capacity of 20 kN and
l from
design tension capacity of 15 kN. The lir|trl
rrce in All dimensions are in
rength
interaction equation between of forces as per
limit state method of IS 800:2007.
_N_
llt due 09. In the above problem the total resultant forcc
dis in the center bolt (Bolt No.5) is KN
stance

HvderabadlDelhilBhopallPunelBhubmewiBensalurulLrcknowiParmlchenmilvijayamdalvizg
lrtrupari lGuoq,,.l Kur"pdyfiyd)
ACE 46 Steel Suucnres
nogineerlngecademv

10. For thefillet welded joint shown in the figure,


the direct vertical shear stress on the weld is
30 MPa and the maximum bending stress is 01. (b) 02. (a) 03. (b) 04. (d) 0s. (c)
120 MPa. For what strength should the weld 06. O) 07. (c) 0s. (1s6.2) 0e. (20)
be designed?
10. (c)
Fror{t Vis,s Side \fiew

(a) 80 MPa (b) 110 MPa


(c) 126 MPa (d) 1so MPa

-)

HydenbadlDelhi jBhopallpgnelBhubaneswlBengalmllrcknowlPatnlChemilVijayamdalVizg lTirupati lGmurl KukaDallvGlvd)


Tension Members
l. Introduction: 2. Types of Cross-sections:
. The tension members can have a variety
Tension members are linear members in which of cross sections. The single angle and
axial forces act so as to elongate (stretch) the double angle sections are used in light
member. A rope, for example, is a tension roof trusses as in industrial buildings. The
member. Tension members carry loads most tension members in bridge trusses are
efficiently, since the entire cross section is I
made of channels or sections, acting
subjected to uniform stress. Unlike compression individually or built-up.
members, they do not fail by buckling. . The circular rods are used in bracings
Ties of trusses, suspenders of cable stayed and designed to resist loads in tension only.
suspension bridges, suspenders of buildings They buckle at very low compression and
systems hung from a central core (such
are not considered effective.
buildings are used in earthquake prone zones as
Steel wire ropes are used as suspenders in
a way of minimising inertia forces on the the cable suspended bridges and as main
structure), and sag rods of roof purlins are other
stays in the cable-stayed bridges.
examples of tension members.
Tension members are also encountered as
bracings used for the lateral load resistance. In
X type bracings the member which is under
tension, due to lateral load acting in one
direction, undergoes compressive force, when
the direction of the lateral load is changed and
vice versa. Hence, such members may have to
be designed to resist tensile and compressive
Cross Stalons a[Tenrion Nen&n
forces.
3. Net Effective Sectional Area (Ao) or (Ao.1):
A,: Ae - Sectional area of bolt or rivet holes
a) Plate of flat with chain Bolting:

bo
I
S*spew{e,rl
Erifrling co

da

e!
(Holediameterd" &n=31
Net effective sectional area of plate or flat
Tensian Mea$en ift Struetfiet with chain bolting along section a-b-c-d-e
A" =(B-nxd")xt

HvdembadlDelhilBhopallPmelBhubaneswlBensalmlLucknowlPatnlChemilvijayawadalvizg lTtrupad lt*i4tlyfltd


lGunnrrl
t"G,"rtffi^-t*t :48 : Steel Stuctures

b) Plate or flat with staggered (Zig-Zag)


Bolting:

A plate of 200mm wide and I2mm thick is


-r as tension member is connected to connect
1

T
plate using MlS bolts as shown in figure'
f'
I
I

critical net sectional arca of plate is


1

B
!
1

I
!
b Hole diarneter ilo=?0mm
.L ,j*TT, -j
gstoto
T
I
(Hole diameter do & n=3) t
;60rnm
I
I
Net effective sectional atea of plate or flat 2,00mm
t60.ro
I
I
with staggered bolting along a-b-c-d-e
i

J- l3Srnrn
,?Smrn
P,2 P,, )
' +-----"-
-*1 '

n, =[n-nxdo + lx t
49, 4gr) (a) 1680mm2 (b) 22orrrr1*
(c)2482mm' (d) 2l60mm'
A*: Plat. $*
Gross sectional area of
Sol: Given pattern of bolting is staggered,
A,,: Net sectional area of Plat. C .f, section may fail along l-2-3, 4-5-6-7, 4-5-
B: Width of plate (.>.
v 3.4-5-2-6-7
n: Number of bolts t. "' L4
do: Diameter of bolt hole I

t: Thickness of plate
I -fs
2{' I
I

l' .l
I 96
A steel plate is 300 mm wide and 10mm thick'
An unfinished bolt of M18 is driven in to it. The
net sectionalareaof the Plate is Net sectional areaalong section l-2-3 (Chai
(a) 3000mm2 (b)2820mm2 bolting)
Net sectional arca of plate - A': (B-nxd") t
(c)2800mm2 (d)1400mm2
An: (200 - t x 20) x 12 :2160 mrrJ
Sol: Diameter of bolt hole do:18i2:20 mm
Net section" Net sectional area along section 4-5-6-7
I,r*i,T;,i; l;** (Chain bolting)

Net sectionalareaof plate: A':1n-n G)l


An: (200- 3 x 20) x 12 :1680 mm'

HvilenbadlDelhilBhopallPunelBhubanewulBengalmllmknowlPatnlChermilvliavawadalvizg lTirupati lGmnrl


_
t

ltrrsiorr Members

Net sectional area alorrg section 4-5-2-3


(Staggered bolting)
Compute the tensile strength of an anglc scction
S USEd / p.') ( ts'\ ISA 100 x 75 x 10mm of Fe 410 gradc ol' steel
A- =l B-nd + '' lxt=l 200 -2x201 '' lxl2
gusset ' \ " 4s, / ( 4x60) connected with gusset plate with fillet wcltl lxrscd
:. The
:2201.25mm' on gross sectional yielding.
(a) 330 kN (b) 622.skN
Net sectional area along section 4-5-2-6-7 (c) 547.8kN (d) 37s kN
(Staggered bolting)

( Sol: For Fe410 grade steel, fu :410 N/mm';


p.' p,' )
A-=l
" t " +-r lxt
B-nxd^+'r
4s. 4s.,
I vt
/
) fr:250 N/mm2, y,,,1: 1.25 & y,,'o : 1.10

( J + 752
lsz
=l 2oo- 3x2o+ ' '' )lxl2=2482mlrr| Gross sectional area of an angle
[ 4x 60 4x60) Ar: (100+75-10) x10:1650 mm"
Design tensile of plate by considering gross
Net effective critical sectional area is along section yielding failure T6,
section 4-5-6-1is 1680 mm2
1650 x 250
1.10
:d, the Limit state desien conceDts
4-5-2- :375x10' N:375kN
'1. Types of failures in a tension member:
a) Gross section yielding failure of the b) Design tensile strength based on net
member (Limit state of yielding in the section rupture (Ton):
gross section)
b) Net section rupture failure of the member
D A Tension member (Flat or Plate)
connected to other member or gusset
(Limit state of fracture or rupture) plate by bolt or weld
c) Block shear failure of the member. Nominal tensile strength of a plate or
flat based on net section rupture
Design strength of tension member (T6):
T*=0.9xA"xf"
a) Design tensile strength based on gross
(Chain
section yielding (TaJ: Design tension strength of member
based on net section rupture
Nominal or characteristic tensile strength
xdo) t
of member by considering gross section
.,.
tdn
_T* _0.9xA"xf"
- \t
mm' yielding -rml rml
^t

Tn* ii) A Tension member (Angle, channel,


tee section etc) is connected to othcr
7
Design tensile strength of member by member or gusset plate by bolt or
considering gross section yielding weld
Tn,
"'- A, x f, Design tensile strength of a nrcntbe:t'
t
d") To"-
- based on net section rupture
mm' Y.o Y,no

|@|Delhi|Bhopal|Pue|Bhubmesw|Bengalm|Lucknow|Parra|ChemaiVijalamda|vize|Tirupati|()rlrtru.|Krrhlr1,.r|ly(||v.t{
IlEngin€cringAcademy
ACE
:50: Steel Sructures

Af A-f.
To" =0.9x--nc-u +Px t"'
Tmt Ymo

/c'\ A lap joint is used to connect two flat plates of


where F = r.4 -0.076" width 220mm and l0mm thickness with chain
[+,.i"i] pattern bolts having bolt hole diameter 22mm for
(t.. \t ) M20 bolts as shown in figure. The yield stress
0.7<B<l;tt-l-lll and ultimate tensile stress of plate are250 N/mm2
\ru T'') and 410 N/mm2 respectively. Partial safety factor
for material governed by yield stress and ultimate
An : Net effective sectional area of a tensile stress are respectively y,no: 1.10 and
cross section ymr :1.25. Determine design tensile strength of
Ar": Net sectional area of connected leg plate by considering net section rapture.

Aso: Gross area of the outstanding leg


w: Outstanding leg distance
t: Thickness of the outstand
L. : Length of end connection (Distance
between the outermost bolt in the joint
along the length or length of weld along
(a) 277.2kN (b) 31s.0 kN
load direction
(c) 4s4.6 kN (d) s02.2 kN
b.: Shear leg distance
Sol: fo:410 N/mm2; TnFl.25
lu fr:250N/mm2; T.o:1.15
Design tensile strength of plate by
F-t
w?
b,:w considering net section rapture T6n
l7-elileilAngle
br:w*wyt
BolhilAngle /{ 0.9x(220 -3x22) x 10 x 410
Angles wilh End Connection Tan =0.9!!t!-t--
4/
, ml t.25
Note: For preliminary design of tension = 454.60x 103 N = 454.60 kN
member 15800:2007 code recommends
following formula for design tearing
strength ofnet section c) Design tensile strength based block shear
T ^ .. A"f"
(the block shear strength at end
Idn =GX- connection) (Tar):
Tmr
a = 0.6 for one or two bolts (n S 2)
: 0.7 for three bolts (n:3)
D For qbear yield and tension fracture:
: 0.8 for four or more bolts (n > 4) A" {
Tuo, =1$!+0.9 x "
= 0.8 for weld lengths ./3y-" Tmr
Where n: number of bolts used at a joint

Hydenbad I Delhi I Bhopal I Prme I Bhubmesw I Bengalw I Ircknow I Panra I Ctremai I Vijayauda I
Viag Tirupati Guur I KukapallvGlvd)
I I
z 52:. Steel Sructues

Limiting or 8. Tension Splice:


Condition for load reversal maximum . It is a joint for a tension member, tension
slenderness splice is provided when Length of
ratio member required is higher than available
A tension member in which length from Indian rolling mills or factory
reversal ofdirect stress due or when two different or same lengths of a
to other than wind seismic 180 tension member have different thicknesses
loadine (or cross section) are to be connected with
A member normally acting filler plate or packing plate
as a tie in roof truss or a . Tension splices are provided on both sides
bracing system but of member joined in the form of cover
subjected to possible of 350 plates.
reversal of stresses ing plate
resulting from action of
wind or earth quake forces
For any other tension Tension Splice
members (other than pre 400
tensioned members) The strength of the splice plates and
bolts/weld connecting them should have
strength at least equal to design load
. The design shear capacity of bolt carrying
A flat tie150 ISF 16 is carrying load reversal due to shear through packing plate in excess of
loads other than wind or seismic loading. Determine 6mm shall be decreased by a factor Bprg
maximum length of flat as per IS800.
(a) 1848.0mm (b) 831.6mm Fpre: 1.0 - 0.0125 x tpkc
(Where tprg: thickness of thicker packing
(c)7794.0mm (d) 1617.0mm
plate)

Sol: A tension member in which reversal of direct


stress due to other than wind seismic loading
l/r < 180
Two plates 8 mm and 16 mm thick are to be
joined by a double cover butt joint wiqh packing
Limiting slenderness ratio: l^*/rr;ns 780 plate and cover plates of 8mm thickness. The
l^*:Maximum length of a flat tie effect of packing on the design shear strength of
bolt is multiplied with a factor
rr;,:Minimum radius of gyration of flat (a) 0.10 (b) 0.e0
(c) 0.20 (d) o.8o
t6
= 4.62mm
Sol: For double cover bolted connection between
8mm and 16mm thick, a packing plate of
Maximum length of flat tie l** Smm thick is to be provided.
The design shear capacity of bolt carrying
: 180 x 4.62:831.6 mm shear through packing plate in excess of 6mm
shall be decreased by a factor pptg

HyderabadlDelhilBhopallhmeiBhubmeswlBengalurullrcknowlPatraiChemailvijayawadalvizs lTirupati lGunurl fuUAallyGfva)


tffi.m.=* :53: 'l'crrsiorr Mcmbers

llcduction lactor on thicker packing The whole area of thc mcntbcr shall bo
on
:
[]pkg: 1.0 - 0.0125 xtpre:1.0-0.0125 x 8 0.9 taken as effective section ratltcr tltittt rtcl
of effective section(whole arcel ol' tltcr
ble (Note: The design shear capacity of bolt is member is gross area less deductiorr lirr
)ry reduced by I0%) bolt holes)
,fa The strength of lug angle and fastonors
ses 9. Lug Angle: connection to gusset plate or any othcr
,ith X{rtfut,4ng!e
attachment should be at lea 0 o/o,
if main member is channel) n the
des force in outstanding leg.
ver The strength of fasteners connection lug
angle to main member shall be atleast =10-
force carried by the
outstanding leg (20 o/o, if marn member is
.4ngle Tiu ttith Lrtg Attglu channel)

In order to increase the efficiency of the orking Stress method concepts


and outstanding leg in single angle tie or
raVe
outstanding flange of channel in channel tie and
to decrease the length of the end connections, L0. Tensile load carrying capacity of a section
sometimes a short length angle at the ends are (P,):
ring
connected to the gusset and the outstanding leg
sof
c
of the main angle directly or outstanding flange A.net : Net area of the cross section
of a channel tie,. Such angles are referred to as
lug angles. Permissible or Allowable stress in tension
ng . Lugangleiss@(or
channel) used at a joint to reduce the
o4:0.6xt
length of connection of heavily loaded t: yield stress for steel
tension member.
. By using lug angle there will be sq4lqg_!4 A tension member is designed for its net
rbe sectional area at the joint
king 11. Net effective sectional area of plate (An.1):
The
is not preferred. Anet : A, - Sectional area of rivet or bolt
th of . for lug angle holes
IS 800:2007 specifications
are: A, : Gross sectional area of a member

a) Plate or flat with chain riveting:


shall as far as possible at the end of the
connecflon

member shall preferably start in advance


of the member to gusset plate
rymg
6mm
be used fo t?
(Hole diarneter et,, & n*-j;

|ffiadDelhi|Bhopal|Pme|Bhubmesw|Bengalm|Lucknow|Patrra]ChemaijVijayawadaivizgiTirupati|(irrlrtrrr|hrll'rr1lr||1(||v.|)|
ACE
lnngineeringecaaemy : 54: Steel Stuctures

Net effective sectional area of plate or flat


with chain riveting along a-b-c-d-e
An., =(B-nxd)xt

i
b) Plate with staggered(Zig-Zag)
Riveting:
I
It
Where, +- !t ------+
*-
,".b T Ar : Net sectional area of the
{ {
B Q \,. $' connected leg
lS'

t l-
Az:
: (x- d -tl2)t
Gross cross sectional area of
{flolediameterd &n=3) unconnected or outstanding leg
: o-tl2)t
Net sectional area of plate or flat with
staggered riveting along a-b-c-d-e
( p,, p., ) 1+0.33&
r'ts'l B-nxO*'t a-2_lxt
A-.=l A,
\ 4g' 4g, )
Area of a leg of an angle
Ar: Gross sectional area of plate : (length of leg - ll2 x thickness of leg) x
Anet : Net effective sectional area of plate thickness of leg
B: Width of plate or flat
d) Pair of an angles placed back - to -
n: Number of rivet holes along section back connected by one leg of each angle
d: Gross or effective diameter of rivet (or by the flange of a T- section) to the
t: Thickness of plate or connected member same side of a gusset plate (with tacking

p1 or p2 : staggered pitch (measured parallel


rivets):
Net effective sectional area An.1: ,{1 * k2A2
to the force)
Ar : Net sectional area of connected legs
gy or g2: gauge distance (measured (or Flange of T section)
perpendicular to the force direction) :
Az Gross area of outstanding legs (or
ln zig - zag riveting, the net cross sectional web of T section)
area along a chain of rivets is increased by
p'xtl4gfor every one inclination P*ir*f nnglex ort
same side of gurwt

s/ c) Single angle connected by one leg only: T*efting ti:tet


Net effective sectional area of single
angle
Anet: A.1 * k1A2

CIussdFfal€

HydenbadlDelhilBhopallPmelBhubanemlBurgalmlLucknowlPatnlChennail\njayamtlalVize lTirupati lGmtrl Kuka4anyHyd)


-*

Acadcmy :55 : Tension Membcrs

ru 54, I member should be taken as effective. [For


l!.
' -
-t-----------i
(5A, + Ar) IS specifications refer para 9l:
A.net : gross sectional area- Area of rivet
t+ 0.242
Ar
holes.
area of web of T section (depth of T - :
-
ion thickness of flange) x thickness of 13. Design of axially loaded tension member:
Design Procedure:
outstanding legs of pair of angles should l.Calculate net sectional arearequired (Ao",)
lucked by rivets at a pitch not exceeding :
(Ao.),.q Axial tensile load lpermissible
axial tensile stress

double angles or T sections carrying Axialtensileload P


placed back to back and connected (An", ),"0 =
Permissibleaxialtensilestress our
h side of a gusset or each side of rolled
scction (with tacking rivets):
,' l'f-ective sectional area (An.t) 2. The net area increased by 25% - 40oh to
(Gross sectional area- area of rivet compute the gross cross sectional area
holes) calculated by Ag.u,,
Pair of angles on 3. Choose a trail section from steel table
having sectional area is approximately
same as Agross oS per step (2)
4. Calculate the available net effective
sectional area of trail section (A.n",)uuo

5. The trial section will be suitable and safe


if (A,o)^u" > (An"t)."q as per step (l) above
6. Calculate Number of rivets required,
n- Axialtensileload P

that the angles or the T sections are RivetValue P. Rv


veted along their length at a pitch not 7. Check the slenderness ratio when
ing I m.
reversal of load may occur as per IS 800

pair of angles or T sections placed


- back are not tack - riveted. then 14. When members of different thickness are
under (d) and (e) above do not spliced, packing plate or filler plates may be
and each angle or T-sections is used to bring the member in level
as single angle or single T-sections
to one side of the gusset as a plato

lug angles are used to connect an Tension Splice


member, the whole area of the
ACE : 56: Steel Strucnres
EDSioc€ringAcadeqY

. If rivets or bolts carrying calculated shear


,t"t, through plate is greater 6mm
- packing by above number
should be increased
required by nominal calculations by 2'5 Figrue I
p.i..", for each 2mm thickness of packing'
the additional rivets or bolts may preferably
be placed in an extension of the
packing'

01. Which one of the following is not a tension


member?
(a) Cable (b) Bar
(c) Boom (d) Angle
(u)pz >2sd (b) p2 t J4 ed
has been used
02. Steel of yield strength 400 MPa
- (r) p2 ,4gd (d) p > agd
i" structure. What is value of the maximum
allowable tensile strength weather as per ity of single angle ISA 100 mm x
MPa 05. The capaclty
IS800:1984 is

03. Two equal angles, each being


ISA.100 rnm x
- are placed back-
iOO mm of thickness 10 mm,
to-back and connected to either side of
a
(a) 333lN (b) 2531d.{
gusset plate through a single row.
of 16 mm
(c) 238 kN (d) 2101(].{
The effective
iiameter rivets in dor le shear'
legs
areas of the connected and unconnected
and 950 06.
of each of these angles are 775mrrf
If these angles are not tack-
to*t ,.rp..tive1y.
pair of
riveted, the net effective area of this
ansles as Per tS8000:1984 is
ratio permitted is.
l;j"i;:;il' @) 3450 mm2
(a) iesser for member 'M' than that
i;;;o;;-* (qzare*'''
member'N'
04. Two bolted plates under tension with (b) More for member 'M' than that
alternative arrangement of bolt holes
ate member'N'
shown in figures I and 2'The hole diameter' (b) Same for both the members
pitch, and gauge length are d,,pandg,
p
(b) Not sPecified in the code
respectivelY.

: :: : ; * nailviFyawadalvizs lTirupari lcunurl


tlydenbad I Delhi I BhoPaI I hne I

i
,fi! ACE
faefuccringAcad€my :57 z Tension Members

07. A member acting as tie in a roof truss or in a I L Consider the following statements, lug angles
bracing system subjected to possible of are used to
reversal of stress resulting from action of I. Increase the length ofthe end connection
wind or earthquake forces tension member as of angle section
per IS 800, where reversal of load is due to II. Decrease the length of end connections of
loads other than wind or seismic loads should angle section
not exceed III. Increase the lengths ofthe end connection
ofchannel section
0l(. The best tension member section will be a IV. Decrease the lengths of end connections
(a) Double angle section on same side of
ofchannel section
gusset plate with outstanding legs are tack
Which of the followine statements are
bolted or welded
correct?
(b) Double angle section on same side of
(a)I&II (b)rr&ry
gusset plate without outstanding legs are
(c)I,il&N (d)r,il&ilr
tack bolted or welded
(c) Double angle section on opposite sides of
' gusset plate with tack bolted or tack
welded 01. (c ) 02. (240) 03. (d) 0a. (c) 0s. (d)
(d) Double angle section on opposite sides of
gusset plate without tack bolted or tack 06. (a ) 07. (350) 08. (c) 0e. (c) 10. (40
nmx
welded 11.
rcr is )
0mm l)t). The maximum length of a tension member
wable
with minimum radius gyration of 20mm
carrying load reversals due to loads resulting
from wind or earthquake forces as per IS 800
is
(a) 3.0 m (b) 4.s0 m
stress
.N' (c) 7.0 m (d) 8.7s m

As
erness
10. In case of
angle section lug angle, their
attachment to the main angle member should
nat of
capable of developing xYo in excess of force
in outstanding leg of the angle, where x is
rat
Compression Members

1. Introduction:
. A compression member is a
structural
member which is subjected to two equal
opposite compressive forces applied
at its in the Euler mode.
. Intermediate columns generally fail by
ends. There are many types of compression
known' inelastic buckling
members, the column being the best
Top chords of trusses, bracing members'
boom is another principle compression
flanges
member in a crane and compression
of built up beams and rolled beams are all of
Design Compressive strength (Pd) a
examples tf .o*ptttsion elements' Columns
are usually thought of as straight vertical
members whose lengths arc considerably
Breater than their
cross-sectiot"
. ttlt::t:h,
ing equal and compression members.
is considered
first. Columns and struts are termed "long!'
or P6: A' x ft6
"short" depending on their proneness to Where A.: Effective sectional area
buckling. lithe tttot it "short", the applied f"6: Design stress in axial
forces *ill cuus. a compressive strain,
which comPression
in the curves
results in the shortening of the strut IS:800-2007 proposes multiple column
direction of the applied forces' Under (a, b, c and d) of figure based on PerrY
incremental loading, this shortening continues Robertson aPProach
until the column "squashes"' However' if the
Jrut is "long", similar axial shortening is 't

observed ottfy at the initial stages


of 0,0

incremental ioading. Thereafter, as the 0.0

the
applied forces are increased in magnitude' .!-'
,i*t b..o..t "Lmstable" and develops a (r,?

deformation in a direction normal to


the 0,.
0.4
loading axis' The strut is ina"buckled' state' aa
to
Buckliing (mainly in members subjected 0.r
.o*pr.rriue forces) behavior - is thus 0
O 05 1 , l't '
chicterized by deformations developed in a
direction (or plane) normal to that of the
Brekliag *rv'es as Per I$&X):!{X}7

loading that Produces it. . Code also recommends following equation


for
estimating design axial compressive stress
(f.o) of axially loaded compression member' it
:: : t"" nrlalvize lTirupati lcub'l Kuk'SalvGlvd)
HyderabadiDelhif Bhopall
considers the residual stress, initial
imperfection and eccentricity of load
fy A principle strut of a roof truss is composed of
Y.o fn .fu two equal angles of ISA 75 x 75 x () mm are
r-cd - r-
ms: ------Fl----T < -
^ Y''o T,'o
connected back to back on either side of l0mm
axial 0+10' -Y I- - thick gusset plate. The cross sectional area of
ng.
ding
l-t each angle is 866mm2 and moment of inertia
O=o.sl t+o(1. -o.z)+tJ (I",:Iw) is 457000 mmo. The distance of centroid

lbv
LI I

of angle from its surface (C,,:Cw) is 20.6 mm.


The minimum radius of gyration is
l,: Non dimensional effective slenderness
(a) 20.6 mm (b) 22.97 mm
(c)29,22mm (d) 14.60 mm
I n2E
I :
Sol: Cross sectional area of strut A 866mm2
Moment of inertia I"":Iw:457000
ofa ry1
Minimum radius of gyration r.in =./U
yield VA
g the
raded
tar Moment of inertia of section about Y-Y axis
'* : Effective slenderness ratio (symmetric axis)
r Iw + Azt)=2r, +2A22
= z(ln
KL: Effective length of a member
Moment of inertia of section about Z-Z axis
K: Effective length constant
(C.G of built-up section and C.G of single
o, : Imperfection factor
section are in a same axis, hence y:0)
Buckling class a b c d Irr= 2(I- + Ay')= 21,,

Imperfection Minimum moment of inertia I^in: Irr: 2I",


0.21 0.34 0.49 0.76
factor (o)
Minimum radius of gyration
Stress reduction factor account for residual r_in ={tli*
stresses: 1=----l ^-.
0+[0' -]"'1"' 2x457000
=22.97mm
T.o: Portial safety factor for against

vield stress
4. Effective length of Columns (KL):
The effective length, KL, is calculated from
on for the actual length (L) of the member,
stress considering the rotational and relative
ber, it hanslational boundary conditions at the ends.
60 Steel Strucnues

Boundary Conditions
At one end At the other end Schematic rePresentation

\l
\'s. Restrained Restrained
L

!r-ttne&'
Restrained
$"dblo"

-$;
aI

$, il
il
\l
I
7Vt

L is the unsu of the compression member


: 61 : Compression Members

Determine design axial load on column section


When the column is effectively held in position ISMB 350 given that the height of column is
and restrained against rotation at one end and at 3.0 m and that it is
pin-ended. Assume
other end is held in position or nor :
fr:250 MPa, fu 410 MPa and E:2x105 MPa
lostrained against rotation, the effective length of (properties of ISMB 350 are F6670 mm2,
Lrolumn is 'K' times unsupported length (L) of the tr:14.2 mm, tw: 8.1 mm, b:140mm, h:350mm,
column, where K is trr:143 mm & r*:28.4 mm)
(a) r.2 (b) 2.0(c) l.s
(d) 0.8
(b) 255.3 kN
@)7a4.23kN
Sol: Given end condition are translations and (c) 766.s kN (d) 383.s rd{
rotation are restrained at one end and
Sol: Design axial load on column Po : f.o.A,
unrestrained at another end (i.e Fixed at one For ISMB350 A:6670 mm2
end and free at other end)
Effective length constant: K:2.0 rrr:l43mm; r,,:28.4mm
h: 350 mm; br: 140 firni ts= 14.2 mm
5. Effective length for angle struts: h 350
For =2.5 >1.2 tr< 40 mrn
bf
-- r40
Section Effective lensth
Buckling class about Z-Z axis - a
In plane Perpendicular
of eusset to susset Buckling class about Y - Y axis - b
(iontrnuous Imperfection factor a : 0.21
Single angle
angles (top or 0.7 L to (Buckling class a)
or double 1.0 L
hottom chord 1.0 L :0.34 (Buckling class b)
angle
of trusses)
Single angle
Effective slenderness Ratio
connected 1.0 L 1.0 L Effectivelength _ KL
with one bolt MinimumRadiusofduration r.in
Single angle
[)iscontinuous
connected Effective length: KL: 1.0 L:3.0 m
angles
with more :3000 mm
0.85 L 1.0 L
than one bolt
KL 3000
are equivalent
Effective slenderness ratio: -
weld r 28.4
Double angles : 105.63
placed on
Non dimensional effective slenderness ratio
either side of
the gusset
0.70 L
Discontinuous olate
to 0.85 1.0 L
ungles Double angles
L
placed on
same side of
the gusset
plate
l": zso(ros.o:)'
--."t'"-'"-1 : l.l8
n2 x2xl}s

HydenbadlDelhilBhopallPrme!BhubanesmlBengalmlLucknowlPatnalChemiivijayamdalVizg lTirupati lGunnttl f,ulrttrlly(llytll


;62: Steel Structures
il$i--ffi^-.*"
0
: 0.5 (1 + o(1" - 0.2) +)"2)

:
0.5(1+0.34(1.18 -0.2) +1.18')
: 1.36

Design strength in axial compression


T
. _ frly*o .jt
"o-o*(o'-l'')o'-t"," 5 I

25011.10 r=
r:o*(r r'-1.18')o' L/d ratio:

: 111.58 N/mm2 3 frly,o 6. Buckling class of cross sections:

Design axial load on column P


: Design
axial strength of column
Pa = f.a.A. : 1 11.58 x 6670 :744'23 x 103 N
:744.23kN

< /<100 mm
A tubular circular column section is having outer
diameter is'..6d'and inner diameter is'd'' The
rt-t,

column is effectively held in position at both ends


and unrestrained against rotation at both ends'
The effective slenderness ratio of column is 200'
The L/d ratio of column is / <40 mm
(a) 200 (b) 100 (c) s0 (d) 0 f-
I

Sol: Effective slenderness ratio


:Y'Lk -- 200 t >40 mm
f
li
Where KL: effective length of column
I
Effective length constant
: 1.0 (for hinged-hinged end condition)
r: Radius of gyration of tubular circular ilnannet,
A"gle, T and Solid Sections
scction

7, Design of axially compression member:

f = f-in
[*)[(r"l-u'] Design Procedure:
. design stress in compression(f.6) in the
;[k"l -,'] member is to be assumed .
'For angles struts f.a= 90 N/mm-
. For rolled steel beam sections
f"a:135N/mm2

Hydenbad I Delhi I Bhopal I Prme I Bhubanesw I


ures
ifnngineerlngAcademy
ACE
:63: Compression Members

. Column with heaw factored load


f.o:200N/mm2
. Effective cross sectional area required for A strut of a roof truss is composed an angles of
factored load P; A.:P/f"6 assumed ISA 60 x 60 x 6 mm are connected to l0mrn
. Select a suitable section to give effective thick gusset plate is subjected to compressive
area required and calculate minimum loads resulting wind or earthquake forces. The
radius of gyration (r",iJ for selected cross sectional area of each angle is 684mm2
section moment of ilertia (Irr:Iw) is 226000 mmo, Iuu is
. The effective length of the column is 360000 mm* and Iuu is 91000 mma.Determine
calculated based on end conditions and maximum length of strut of a truss as per IS800 is
the slenderness ratio is computed (a) 2.07 m @) a.s5 m
(Kl/rni"), which should be less than (c) 2.88 m (d) s.72m
maximum slenderness ratio
. For the estimated slenderness ratio, the Sol: A member subjected to compressive forces
design compressive stress fca and design resulting only from combination with
l"*-l
mgl compressive strength of section P6 which wind/earth quake actions Ur < 250.
llass should be higher than factored load. If not

;_l
bl
I repeat above steps.
tf. Maximum slenderness ratio (Stiffness
Limiting slenderness ratio : l^u*lr^ins 250
l*o,:Limiting length of angle tie

:l Requirement):
The IS: 800 impose the following limitations
r.;n:Minimum radius of gyration of angle tie

bl I on the slenderness ratio of members subjected


to compression t''" ={f
/r.," ={ffi
_ Frooo =ll'53mm
cl
dl
--l
dl
Load Condition
Limiting or
maximum
slenderness
Maximum length of a strut l^*
:250 x 11.53:2882.5 mm:2.88m

bl I

ratio 9. Built up columns or sections:


cl (a) A member carrying (a) Used when rolled steel sections do not
cl
ol compressive loads from
dead and imposed loads
180 provide required sectional area or large
radius of gyration of column section is
I
(b) A member subjected to required to different directions built up
al compressive forces 250 section.
cl resulting only from (b) Usually provided either lacing or batten
-;_--] combination with system for built up column.
wind/earth quake actions
(c) Compression flange of a 10. Lacing and Battening for
built up
beam restrained against 300 compression members:
torsional bucklins (a) The different components of built up
sections are placed in such a way that the
built up section has same radiue of
gyration about both axes. (i.e. r2p = fyy)
Steel Strucures
ACE
"#..
:EEIEnd;;fuAcademv
It
:64:

--

are used for lacing'


-should
;Jciri;- system not be varied tn*
lfnouiftout the length of the member' l
irt. ,r:ngr. laced system on opposite.sides
of the main components should
be in the
L-*-S
NSICL3 Llclls
;;d*ttion so that one be the shadow (/"')
Effective length of lacing member
. i"t t1tgf. tacing (Bolted) , -1":
of the other. I
ii;^ntil should be provided 1t the ;1as . Po, do,lbte lacing (Bolted at ends)
wnere
of the lacing syst . r and at potnts lr= 0'll
lacing sYstems are intemrPted' . For welded lacing In:0'71
sYstem'
For Bolted or welded lacing
L/r"6n < 50 or : 0.1 times (KL/r)o of
as a whole, which ever is
less'
-.*U.t of gyration
Where rftn= minimum radius
of the comPonents of compresslon
member
. il;il"* width of lacing bar in Bolted
connection

Sttuot diameter of
should not exceed 145' the bolt (d) in mm
orNominal diameter

wiatn of lacins bar

. Mioi-o. thickness of lacing bar


o t-i,,: I | 40 for single lryfg
. i;;: 1/ 6o for double lacing
-ca-\.
,l!--t5uo ACE
CS :f)f@ :65: CompressionMembers

Where,
/:length of the lacing bar
. The lacing should be designed to resist a A built up column consists of ISMC 300 channels
transverse shear (V) of placed back to back at a spacing of 200mm and
carries working axial load of 1500 kN, the double
Y :2.5oh of design axial column load lacing provided with an angle 45o with
(LSD Principle) longitudinal axis. As per IS 800:2007 lacing
Y :2.5o/o of axial column load member should be designed to resist desien axial
(WSM Principle) load of
The lacing should be designed to resist (a) 22.5 kN (b) s6.3kN
additional shear due to bending if the (c) 19.9 kN (d) 3e.8 kN
compression member carries bending Sol:
For single lacing, the force (Design Design load on column P: 1.5 x 1500
compression or Design tensile) in each
:2250 kN
lacing bar,
Transverse shear V:2.5x22501100 : 56.25 kN
F_
Nsin0' V
Force in lacing member F =
V Nsin0
F = -- for double lacing (N = 2)
2sin0 (N:4 for double lacing system)
V 56.25
F =_ for double lacing (N = 4) r=_=19.88kN
4sin 0 4x sin45
. The effective slenderness ration of laced 12. Batten for built-up columns:
of column should be increased by 5% (a) General Specifications:
(IS800:2007 specification only) . The no. of battens should be such that the
lon member is divided into not less than three
ion parts longitudinally (i.e., minimum
A built up column consists of ISMC 450 channels
4 batten plates or
minimum two
lted intermediate battens and two end battens)
placed back to back carries factored load of2500 . Flat plates are used for battens.
kN, the single lacing provided with an angle 450 . Effective length of
battened column
with longitudinal axis should be designed to should be increased bv 10%.
lransverse shear as per IS800:2007 of
(a) 15.0 kN (b) 22.skN (b) Design Specifications:
(c) 62.5kN (d) 44.1 kN . Spacing of battens 'C' is such that, the
slenderness ratio of the lesser main
Sol: Design load on column P: 1.5 x 1500
C
:2250kN component, " > 50
r-;o
or 0.7 (KI/r)" of

Transverse shear V: 2.5x25001fi0 the member as a whole about Z-Z axis


(parallel to battens), which ever is less.
:62.50 kN

Hydenbad!Delhi!BhopaliPue lBhubmesw BengalmlLucknowlPamajChemaiiVljayawada Vizg lTirupati lGrrutrrr'l Krrlrltprffriifr,ff I


Steel Structtues
ffit-tffi^-t-t' :66:

I
N: No. of Parallel Plates of battens
I
:2 inthe above figures.
I
Da'
tro
OG
OG
dc
Thickness of batten, t>hl 50
DO (rf
ore
--l L : length of batten Plate
a
Effective dePth of batten
c d> 3al4 for intermediate batten
d>a for end batten
d> 2b for any batten
[eo o9
ct6 Over all dePth of batten (D)
loo E6
lao
!o6 OG D: Effective depth of batten (d) + 2x
L Ib

Fb -{
13. Axial compressive strength (P.) of a
member:
P.: A x ou"
Where
v
l**.-- a A: Effective sectional area of a member'
o^": Axial allowable compressive stress
Where, -*l
r"-in: minimum radius of gYration IS800:1894 uses Merchant Rankine
of comPonent. formula to calculat€ oac!

S : transverse distance between centroids f f.,


of bolt group or rivet group
o* =0.6;F<0'6f., r
1.u -f.f,J""
C: Spacing of battens
/6 : transverse distance between centroids
Where.
of inner end bolt group or rivet group ! : Yield stress of steel
Battens should be designed to carry f..: Elastic critical stress in compression
bending moment and shear forces arising
from a transverse shear of
: n'E I ll(Lhl2
Y :2.5o/o of design axial column load E : Young's modulus of steel
(LSD PrinciPle)
KLh: Slenderness ratio
Y :2.5oh of axial column load
(WSM PrinciPle) effective length of member
radius of gYration of member
----+- Longitudinal shear on batten, Vr: VCNS
n : a material factor (n : 1'4 for steel)
Moment on batten, M:VC/2N Radius of gYration
--r-
J[-/ AJ = .fto-.nt of inertia I
Where, Atea
,=
' tdalviz"c lTirupati lc'nt'l KukaganvGlvd)
14. Effective length(KL) and allowable axial 16. Cased Columns:
compressive stress For angle struts, the lt is necessary to encaso lltc trtcn-rbers of
specifications as per 15800:1984 are as steel framed building in cortct'c:lr.: to meet
follows: architectural appearance and itlso ittcrease
fire resistance and check corrositttt ol'steel
Effective Allowable members
End Condition length axial For designing of cased column, tltc e trlirc
(Kt) compressive load is assumed to be taken steel socliolt
stress (o,.) only and encasement is taken incrctsittg
(a) For discontinuous members the stiffness of the column
For single rivet 1.0 L 0.8ou,
;X
or bolt tions of encased columns:
lCe
For double rivet
. The member is of symmetrical I-shape or a
I
{ Double 0.85L oac
bolt & weld single I beam, or channels back to back with
ft) For Continuous members or without flange plates.
of For single or 0.70L . The overall dimensions of the steel section do
double angle to oa" not exceed 750 mm x 450 mm, the large
0.85L dimension being parallel to web.
. The load carrying capacity of encased column
)ef, t5. Design of axially compression member should not exceed 2 times the load permitted
s Design Procedure: on an uncased column.
. Stress in axial compression(o*) in the . The minimum width of solid casing is
ankine member is to be assumed bo + 100 mm, where bo is the width of steel
. For angles struts o,.: 65 N/mm' flange of column in mm.
. For I beam sections on": 80N/mm2 . The radius of gyration for encased column
. Built-up columns o..:110N/mm2 (about YY-axis) is given by r"v.
. Effective cross sectional area required for r'':0.2 (b"+ 100)mm
axial load P; A":P/o.. assumed Where.
. Select a suitable section to give effective bo: width of steel flange in mm;
area required and calculate minimum r'is taken as that of the uncased
radius gyration (r-i.) for selected
of section.
section
. The effective length of the column is
calculated based on end conditions and
the slenderness ratio is comPuted
(KL/r*o), which should be less than
maximum or limiting slenderness ratio
. For the estimated slenderness ratio, the
r
allowable stress in axial compression o..
:el) and axial compressive strength of section
P. which should be higher than axial
column load. If not repeat above steps.
rftt ACE
lfl Ein"gncering ecademv :68: Stuchrres I

-- (c) Buckling strength based on/the net area


the section and percend elongation
Class Room Practice Questions ultimate load
(d) Compressive strength based on effective
slendemess ratio and gross sectional area
01. For a circular column section having its ends of the section
hinged, the effective slenderness ratio is 200'
The ratio of Ildof column is-
05. A strut member used in a roof truss cons
of equal angles of ISA 100 mm x 100 mm x
02. The stress reduction factor for an axially 10 mm thick are placed on either sides ol
loaded compression member depends upon 16mm thick gusset plate. The cross secti
which of the following factors of angle section is 1903 mm'
atea
L Residual Stress Moment of inertia isl,,:I,":177 x 104 mm
II. Initial crookedness then the minimum radius of gyration is-
III. EccentricitY of load
IV. Type of cross section A central column of steel water tank
06.
of above factors, the influencing factors isiare
(b)I,[&ilI composed of two equal angles
(a)I&il ISA 100 x 100 x 10 mm used in the form
(c)I,il,n&IV (d)III&IV
star as shown in figure. The cross sectio
area of each angle is 1903 mnt' and major ant
03. Match list -I (Axially loaded member) with
minor momertt of inertia of an each angle i
LisfII (slenderness ratio) and select correct i) = l, :177 x104 rffn4, \u: 282.2 t 1-0' 1T
answers using the code given below list
unA t,":Zt.g " 104 mma respectively'
List -I distance of the angle from its surface i
A. For compression members carrying Dead (Qr': cyy:28.4 mm)' The minimum radius
load and live loads
gyration of central column is
B. For members carrying compressive Due to
(a) 19.4 mm
wind or seismic loads onlY
i
C. ComPression flange of beam (b) 30.6 mm
(c) 38.5 mm
I
List-II
I

1. 180 2.300 3.250 (d) 44.6 mm

AB C AB C
I
I
(a) t23 (b)2 1 3
li (c) 231 (d)13 2 07. Which of the following statement is/ are
i' in case of design of lacing sYstem
04. Which one of the following is the most 1. Angle of inclination of lacing bar with
critical set of consideration in the design of longitudinal axis in between 40o - 70o
rolled steel column carrying axial loads 2. The slenderness ratio (I) of lacing
(a) Percent elongation at yield and net should not exceed 145
sectional area 3. Lacing system is preferably used
(b) Critical bending strength and axial yield axially loaded columns
strength of the material
lTirupati lGmurl
HydenbadlDelhilBhopallPunelBhubmeswlBengalurullmlmowlPatralchemailvijavawadalvizag
'uctures
:69: Compression Members

t area 4, Flats, angles, channels and tubular section 12. Consider the following parameters with
rtion at are used for lacing regards to slenderness ratio of a compression
(a) 1,2,3 and4 (b)2,3 and,4 member:
:ffective (c) I and 2 (d) 1,2 and4 1. Material
naI area 2. Sectional configuration
0tl, A Built up column is connected by a single 3. Length of member
lacing system with 45o with longitudinal axis 4. Support end conditions
and subjected to a service load of 1000 ld{ On which of these parameters does the
0mmx due to dead load and live load combination. slendemess ratio of a compression member
sides As per limit state design of ISg00:2007 the depend?
iecti lacing should be designed to resist a (a) 1,2 and 3 only (b) l, 3 and 4 only
,m' transverse shear of ld\f (c) 2, 3 and 4 only (d) I, 2, 3 and 4
l0a mm
s fllm t)t). Find the design axial force of lacing member 13. In laced columns, end tie-plates are provitled
for problem No.08 to
tank (a) 17.68 kN (b) 35.60 kN (a) Check the buckling of column
les (c) 13.36IcN (d) 26.s2 kN (b) Keep the column components in position
form (c) Check the distortion of column sections at
iectiona 10. A column is effectively held in position and ends because of unbalanced horizontal
Lajor restrained in direction at one, other end is held force from lacings.
angle ir in position but not restrained against rotation. (d) Prevent rotation of elements.
lOamm If the actual length is L, the effective leneth
ty. KL is
rface i (a) 0.67L (b) 0.s0L 1. (50)02. (c) 03.(d) 04. (d) 0s. (30.6)
:adius (c) 1.00L (d) 2.00L
(c) 07. (d) 08. (37.5) 0e. (d) 10. O)
I l. Through which of the following responses 1. (d) 12. (c) 13. (c)
may a steel tubular hinged strut fail?
l. Compression 2. Bending
3. Overall buckling 4. Torsion
5. Skin buckling
(a) 2,4 and 5 (b) 1,2 and,3
(c) 3,4 and 5 (d) 1, 3 and 5

afe

with
700
:ing

rsed

Hvdenbad I Delhi Bhopal I Pme i Bhubanem Bengaluru


J
I llucknowl Patu I Cheruni I vijardwada lvizg lTirupai cuuu
I | 4,Jkatrally(Hv )
oofu
1to,rC
I

lffiflf
rr,ud
|.*,
I'iJ rlfr't '
*'Co,IumnBases '

&
Column SPIices
1. Intioduction:
The design compressive stress in
a concrete
3. Slab Base (ConcentricallY
Loaded
iooting ii much-smaller than it is in
a steel
Columns):
columi. So it becomes necessary that
a
below . Fot a purely axial load, a plain square
toituUf. base plate should be provided steel plate or a slab base attached
to the
from it
the column to distribute the load column is adequate'
.".tfyto the footing below' The main
function of the base plate is to spread
the
area and
column load over a sufficiently wide
stressed'
keep the footing from being over
Steel colunu
2. TyPes of Column bases:
Base Plate
steel
For a purely axial load, a plain square
plate or a slab attached to the column
is
plates will
adeqoate. For small columns these Assumed P$s sru'e dishrbutlon

b. ,hop-welded to the columns, but for larger under base Plate

columns, it may be necessary to


ship the
plates separately and set them to the
correct
the columns
elevations. For this second case
anchor bolts
are connected to the footing with
have
that pass through the lug angles which
When there
been-shop-welded to the columns'
vertically
is a large moment in relation to the
be required'
applied load a gusseted base may
from
This is intended to allow the lever arm (*Iuwn base
to give
the holding down bolts to be increased
the base
maximum efficiency while keeping ,*.Ur
plate thickness to an acceptable minimum'
. Slab base
. Gusseted base
,lL
:71 : Column Bases and Colurrrrr Splices

Design procedure:
o Assume a suitable grade of concrete. The
While designing, for a steel column ol' l,t.,tlO
bearing strength of concrete is 0.45 f.r
srade steel base plate resting on a oon(,loto
(Note: A reduced value of 0.45 f.r is used
pedestal of M25 grade, the bearing strenglh nl'
against maximum 0.60 of as f,k concrete (in N/mm") as per IS:456-2000 is
recommended by the code)
(a) I 1.25 N/mm' (b) 9.00 N/mm2
. Area of slab base (A) (c) 15.00 N/mm' (d) 25.00 N/mm2
factored column load (P)
Bearing strength of concrete (0.45 f"u ) Sol: For M25 grade concrete f"u:25 N/mm'
Maximum allowable bearing strength:0.45 ik
. If square base plate is provided
: :1
0.45 x25 1.25 N/mm2
Side of square base plate : L: B : JA
If projections of base plate beyond the column
faces are a &b are kept equal A slab base of size 500mm x 500mm is to be
provided below a column section ISHB250
: (Thickness of flange 9.7mm) with width of
column flange 250mm flange supports a design
B: Width of base plane in mm column load 2000 kN. The yield and ultimate
a : Bigger projection of base plate beyond tensile strength of steel are 250 MPa and
410 MPa respectively. The partial safety factor
against yield and ultimate tensile stress are
column in mm ymo:1.10 and y,65l.25 respectively.
The thickness of slab base is
D: Depth of column section in mm
(a) 9.70 mm (b) 25.82 mm
br: Width of the flange of column (c) 33.07mm (d) 31.02 mm
tr: Thickness of column flange^
w: Upward pressure in N/mm'on underside Sol: f,:410 N/mm2 & &:250 N/mm2
of plate assuming a uniform distribution.
y.o:1.10 & y^1:1.25
*=l A1 Intensity of upward pressure
A.1 :
Area of slab base plate is provided w: P/Provided area of slab base
o The thickness of the slab base (t.) for I, H, : 2000x 103/500x500 : g N/mm2
channel. Box section
Bigger projection of slab base
2.5w (a2 - 0.3b2 )y."
t.= ,tr a : (L-D)12 : (500-250) l2:l25mm
. Holding down 2 or 4 in number and of Smaller projection of slab base
20 mm diameter are usually provided, b: (B-b i I 2 : (5 00-250) 2: t25mm
I
when base is subjected to only axial
compressive load, two bolts will be
enough
:72: Steel Structures

L: Depth of steel column * 2x


thickness of gusset plate + 2 xlegwidth
*
ofgusset angle 2 x minimum
overhang (for bolted or riveted

'*-*i:0 rnm*
* connectlon)

Minimum length of base plate (parallel to


50Omm the web)
L: Depth of steel column + 2 x thickness
Thickness of the slab base of gusset plate + 2 x minimum
overhang (for welded gusseted base)
[s= 2sw (a2 - 0.3b'z)?
Width of base plate (Normal to the web)
Areaof baseplate A
B_
Lengthof basePlate L
= 3l.02mm ) ti
Hence thickness of slab base
t.:31.02mm cohrm

Gusseted Bases: Gusset plate

. When there is a large moment in relation


to the vertically applied load a gusseted Gusset angh
base may be required. Futlation bolts
. Column with gusseted bases, the gusset
Base plate
plates, angle cleats, stiffeners, fastenings,
etc., in combination with the bearing area
of the shaft, shall be sufficient to take the
loads, bending moments and reactions to Corcrete pedestal
the base plate without exceeding specified Riveted or Bolted Gusseted Bflse
strength.
lt
Design procedure:
i
. Assume a suitable grade of concrete' The
bearing strength of concrete is 0.45 f.t
I
Note: A reduced value of 0.45 f.r. is used
l,
rl against maximum of 0.60 tt as

I
recommended by the code)
I

,-c.: Frotrtvies
. Area of slab base
Ft>
:(_ , _--i
factored column load (P) 1;1
(n)=
Bearingstrength of concrete(0.45 f"n )

Minimum length of base plate (parallel to


the web)

HvdembadlDelhilBhopallPunelBhubaneswarlBengalurulhcknowlPanalCherrailviiayawadalvizag lTirupati lGrmuul

I
fi ACE

5.
Column Bases and Column

Column Splice:
. A joint is required in the length of column
Gasset member is called column splice.
Plate . Adopted when the length or height of the
column is required more than tlie tsength of
column section is available from rolling
mills or factorv.
y' \n""" In case of multistoried building the
section column required for various floors
may be different.
Gusseted Base
Column splices are designed as a short
n, - Upward pressure in N/mm2 on underside column.
of plate assuming a uniform distribution Column splices are normally located at
under axial load. section just above the floor level (h/4 from
P floor level)
*=
Ar
. (iusseted base keeps the base plate Sp[ce
Plets
thickness to be minimum.
. 'l-he thickness of Gusseted base is
computed by equating the moment at
critical section (i.e. at root of gusset angle)
and equating to moment of resistance of
base plate.
. l)esign bending moment due to upward Steel
ColilrE
pressure per mmwidth at critical section
M:w c2/2
. l)esign bending strength base plate at the
Specifications of column splice:
critical section Mo: 1 .2 frZly^o . When the ends of compression member
. l'hickness of gusseted base t = IZISY are faced (machined) for complete bcaring
"lfv
over whole area they should be splicr: Lo
Where,
t: :cantilever projection of base plate whole the connected membcrs accurately
(beyond the root of gusset angle in
case ofbolted gusseted base)
in position and this is tension if any
f, = yield stress of steel in N/mm2 bending pressure.
t = Aggregate thickness of base plate
and thickness of gusset angle for When such members are not faced
bolted or riveted gusseted base and (Machine) for complete bearing splice
the thickness of base plate for
welded base plate should designed to transmit all forces to
which they are subjected
z74z
Steel Structures {

W: the
B : the or base in mm
ced end (if anY)
il: the
of the column in mm
be less
The cap of base plate should not
6. Types of Column bases: 1.5(d";7t mm in length or diameter
iili.. ,yp.t of column bases are usually used

. Slab base 8. Design of Gusseted Base:


. Gusseted base Design Procedure:
. Grillage foundation . Atto*. a suitable grade of concrete'
permissible bearing stress of concrete
7. Design of Slab Base: taken as 0.25 f"t
Design Procedure:
A.tri.. a suitable grade of concrete' T1'ls Required area of base Plate
(A)
permissible bearing stress of concrete
-
Axial load in thecolumn
taken as 0.25 f.t Permissible bearingstress in
Required area of base Plate
(A) .
Axial load in the column (parallel to
[= in concrete
. Minimum length of base plate
Permissible bearing stress the web)
The permissible bearing Pressure L : Depth of steel column * 2 x
of gusset Plate + 2 x leg width
.

ts
(or compressive stress) in concrete
4 N/mm-. gusset angle * 2x mtmmum
base plate as
The thickness of a square slab (bolted connection) i, t
per IS800-1984 is Minimum length of base plate (larattet
to the
L: Depth of steel column+.2 x thi
of gusset Plate + 2 x mrmmum
base
ovJrhang lfor welded gusseted
rv : Pressure on underside ofbase
plate
web)
Width of base plate (Normal to the
o6,: Permissible bending stress in slab base (B)
A
Areaof baseplate
: 185 N/mm2 for all steels D_
" - Lengthof basePlate L -
a: Bigger projection of base plate beyond The thickness of Gusseted base
column in mm computed bY equating the moment
b:Smaller projection of base
plate beyond .ii it"f section (i.e. at root ofgusset angl
and equating to moment of resistance
column in mm
o The thickness of a square slab base Plate
per IS800-
under a solid circular column as
1984 is
ctures 75 Column Bases and Column Splices

. l'hickness of gusseted base t = c-/jI' Classroom Practice Questions


mm
V oo,
any)
Wlrcre,
c Cantilever projection of base plate 01. Consider following statements regarding
ot)s : Perrnissible bending stress in slab base gusseted base
t - Aggregate thickness of base plate and L It is considered to be a pinned base
thickness of gusset angle for bolted or II. The gusset material used increases the
riveted gusseted base and the bearing area consequently result in
thickness of base plate for welded smaller thickness of the base plate
base plate. III. The gusset material used supports the base
plate against bending and consequently
r). ( lrillage Foundation:
results in smaller thickness of base plate.
Which one of the followins statements is/are
correct?
Pipe seperator (a)I&il (b)rr&m
'top tier beams (c)I&ru (d) only II is correct
Bo$on tier beaas
02. While designing, for a steel column of Fe250
ickr Grillege Footing grade, a base plate resting on a concrete
pedestal of M20 grade, the bearing strength of
idth
concrete (in N/mm') in limit state rnethod of
Adopted when columns carry very hear,y
loads and the bearing capacity of the soil design as per IS:456-2000 is
is very low.
It consists of two or more tiers of steel 03. A 16 mm thick plate measuring 650 mm x
420 mm is used as a base plate for an ISHB
beams placed one above the other at right
angles to each other and embedded in 300 column subjected to a factored axial
d base) concrete.
compressive load of 2000 kN. As per
IS: 456-2000, the minimum grade of concrete
web)
Pipe separators are used to keep the that should be used below the base plate for
(B) grillage beams properly spaced.
safely carrying the load is
The distance between edges of adjacent
(a) M15 (b) M20
flanges shall not be less than 75 mm.
(c) M30 (d) M40
a Minimum cover of concrete is 100 mm.
a Grillage beam is designed for moment, 04. A of 500 mm x 450 mm is to be
base plate
shear and web crippling
provided below the column section ISFIR
300@576.86 N/m (Width of flange is 250 rnnr
and thickness of flange is 11.6 mm). ll' tlrt.
bearing pressure from the concrete bckrw tlrt.
base plate is 9.0 N/mm2 and yield strcss ol
plate is 250 Mpa. The minimum thiclirrt'ss r'l
base plate is required as per lS l{(X):,'(X)/
is mm
HydenbadjDelhilBhopallPrmeiBhubmeswBengalmlLucknowjPatrnlChemiiVijayawdalVizglTirupari l(irrrrrrrr l(rrh,rrp,rltltttq,lr I
, ACE
lf'ngineeringAcadcmy :76: Steel Stuctures

05. Which one of the following plan views of a 07 . A column base is subjected to moment, if the
gusseted base plate will result in minimum intensity of bearing pressure due to axial load
base plate thickness? is equal to shess due to moment, then the

T
r.-- flff---l e_H*,, bearing pressure between base and concrete is
(a) Uniform compression throughout

G)
I i
110
(b) Tension at one end and compression at
6100
fita other end
Tt
T (c) Uniform tension throughout
I (d) Zerc at one end and compression at other
* end
I
I
-{. 500
ftlll 08. In case of axially loaded column machined for
I
:or ilH
T
I full bearing, the fastenings connecting the
T column to base plate in gusseted base are
r*- ffi --* designed for
(a) 100 % column load
(b) 50 % column load
06. Match List -I (column base) with List-II (its (c) 25 o/o column loa
application) and select correct answers using (d) Erection load only
the code given below list
List -I
A. Grillage foundation
01. (b) 02. (e) 03. (b) 04. (26.3) 0s. (b)
B. Gusseted base
C. Slab base 06. (c) 07. (d) 08. (b)

List-II
l. Lightly axial loaded steel column
2. Heavy loaded steel column to be rested on
weak soils
3. Eccentric loaded steel column

A B C AB C
(a) I 2 3 (b)3 21
(c) 2 3 r (d)2 1 3

HydenbadlDelhilBhopallPrmelBhubmeswlBengalmllmknowJPatnalChemailVliayamdalVizg lTirupati lcmtrl


the
oad
Beams
the
eis
l, Introduction: criteria for Limit State of collapse for steel
lAt A structural member subjected to transverse beams. Steel beams would also become
louds (loads perpendicular to its longitudinal unserviceable due to excessive deflection and
nxis) is called as beam. Beams mav be this is classified as a limit state of
ther r:knsified as serviceability
. Floor beam: A
major beam of a floor
system usually supporting joists in 3. Flexural behaviour of steel beams:
lfor building. If a flexural member is progressively loaded,
the
. Girder: In buildings, girders are the same it deflects and the curvature of such bending
afe as floor beams also a major beam in a varies along its length. Initially the beam is
structure elastic throughout its length. Let us consider a
. Grit: A horizontal member fastened to small portion of the beam at a point A as
and spanning between peripheral columns shown in Fig. where the curvature is p. If we
of an industrial building to support wall consider a small segment of the beam at A
cladding. then the variation of the strain across the
. Joist:A beam supporting floor depth of the member could be found out
construction but not major beams. geometrically as e: zlp
. Lintel: Beam member used to carry From above Equation, the strain at any fibre is
wall loads over wall openings for doors, proportional to its distance'z' from the neutral
windows etc. axis. This is obtained from the assumption that
. Purlin: A roof beam, usually supported plane sections which are normal to the
by rooftrusses longitudinal axis before bending, remains plane
r Rafter: A roof beam, usually supporting and normal even after bendins
purlins
. Spandrels: Exterior beams at floor level
of buildings, which carry part of floor
load and the exterior wall (\rn4wtB'1.$
. Stringers: Beam supporting stair steps (in
case of buildings.
. Header: A beam at stair well openings.
For each strain 'E' one can read off the
corresponding stress 'f from the idealised
Limit state Design Con stress-strain curve for steel shown in Figure
shown below. (The idealised stress strain curve
l, lfehaviour of Steel Beams: neglects the strain-hardening portion for all
Laterally stable steel beams can fail only by practical purposes). We choose four points l, 2,
(a) flexure (b) shear or (c) bearing, assuming 3, 4 on the stress-strain curve for further
that local buckling of slender components discussion and see how these four points aro
does not occur. These three conditions are the used when a simply supported beam in
subjected to a mid point load.
:78: Steel

extreme fibre of the beam is stressed within


elastic range. The corresponding moment, (M ),
just sufficient to cause yield in the extreme
and is given bY
My:fz Z": frZ"

Where M, is calle d the "yield tnoment", r'e'


t

moment which just causes the extreme fibres


ir-l E
yield.
tl 9, {itt '1 'l

Idcnlised dnsttt'pluslir .!ft'd'.r,r- ntf"itt ct'rrv lir sttel


It is evident from Figure below that once
extreme fibre stresses attain yield stress they
longer take any additional stresses.
Consider the point (1) in above Fig in which the 63>Ey rs=ry I 4
strain Consider the point (1) in Fig'2 in which the ''l-^
I
.t.%
I

o V
-T-
I

ifv da
.
I
lzt.

i_

Where Z":Ily is the elastic section modulus


- !. /t
et,, a il",tr ft--"fl

snain Jtrsss

Strain and stress distribution in plasticrange

When the load and hence the moment is

-d*d Strain and stress distribution in elastic range


increased. the outermost fibre strain tmax near
span of the beam (i.e. point of maximum ben'
moment) would attain a value s&], t3> e, and
is identified as point (3) in idealized elasto
stress strain curve. At this stage the strain is in
When beam is subjected to further moment' Now plastic stage, but extreme fibre stress still equi
let us consider the point 2 in idealized elasto yi.ld .tt*tt/.. W. also note that the stresses ha
plastic stress strain curve. The extreme fibre J

been redistributed to the inner fibres towards


st uin equals yield strain r.9' E*a1s:82= e, and also
neutral axis and these fibres gradually altain
the stress Where,/ is the yield stress' Up
f,: f, stress equal to !. This is shown in above figu
to this stage, ai shown in Fig. 3, the stress and The remaining portion of the beam in the vicin
strain are proportional to each other since the of the neutral axis is still elastic. At this stage
Patn I Chemai I Viiaramda Vizg Tirupati Gmtur
Hydenbad I Delhi I Bhopal I Prne I Bhubaneswar I Bensalm I Lucknow I
I I I I
:79 : Beams

thin ftrorncnt capacity is calculated by considering It is easily verified that for a rectangular
section
(M
v
),j lrulh lhc plastic portion and the elastic porlion. the ratio of the plastic to elastic section modulus
le called the 'shape factor' is 1.5. For I-section the
llporr further loading, the outer fibre strain ratio varies between 1.07 to 1.20 and for most
Irrcrctscs rapidly and attains a stage shown as point practical cases ofI-section this is taken as 1.12.
14) irr idealized elasto plastic stress strain curye. This ratio also represents the ratio between the
At this stage the elastic core in the immediate plastic moments to the yield moment.
vtcirrity of the neutral axis becomes negligible due
Il lltr: spread of plasticity into the fibres near the
4. Classifications of sections:
Itr,rrlltl axis. When the entire cross section of the
nce il Itt rrm gets fully plastified, the curvafure become
Depending on yield moment, plastic moment and
they n rotational capacities, the four classes of sections
Itrlirrity as shown in above figure shows such a
r'toss-scction, which is fully plastified. When the
are the plastic, the compact, the semi compact
+ r,rrlilc beam cross section becomes plastic, it resists and slender sections)
@r- . Plastic section: Cross sections which can
li"
)i
rttry lirrther rotation under constant moment. At this
rlrrgu the beam is said to have developed a,plasfic develop plastic hinges and have rotation
'^T- hlnge'.ln view of this rotation, deflections become capacity required for failure of the section by
1,,
vt,t'.y la link at the plastic formation of plastic mechanism are called
t'M
I Itlttgc. ding moment, at plastic section.
whitrh is known as the Compact section: Cross sections which can
'plnslic moment M'. The moment curvature develop plastic moment resistance but have
inadequate plastic hinge rotation capacity for
tclrrlion of the cross ,.l,ion
of the beam, at the point
rrl' nraximum bending moment. The formation of a plastic mechanism before
curvature
lltltcuses enormously once the moment at the cross buckling are called as compact section
t$f liolr reaches M _. The val:ue of M could be easily Semi compact section: Cross sections in
Pp which the extreme fiber in compression can
rlr,lclruined by taking moment of the total tension
reach yield stress, but can't develop the
llrl compression areas about the plastic neutral axis plastic moment of resistance due to local
lr buckling are called semi compact section.
tge
Slender section: Cross sections in which
llt ,ryrnmetrical sections the neutral axis coincides
elements buckle locally even before attaining
Wllh the centroidal axis and this is not so in the
tu ofyield stress are called as slender sections.
0Hrtc of unsymmetrical sections. However the
)ar
plnttic neutral axis for any cross section (also Rotational capacity
onllctf as "equal shear uxis or equal area axis,,)
rnd tl Plastic-
0uttkl be located using the condition that the mortrcnt
plas F
lcttrion and compression areas must be equal as fv
sint A.: At
I
H
equi o Yicld rnoment
ff is scen that the plastic section modulus (Zp) is
-'s ha =
3lvcrr by
rds t Zp: A"Zc+ AtZt
rttain T'ftc plastic moment capacity
of the beam could
figur bo written as
Cuwahlfc
ricini Mp: Za .fy
age tJ

HvderabadlDelhi]BhopallPunelBhubmeswlBengalmlLucknowlPanralChemailviiayawadalvizg
lTirupati lGunturl Kukarpllly(lry<t)
: 80:
e*g,ffiAcad€mv
o Check for deflection
o Check for secondary failures like
buckling of comPression flange or
CrossSectionswhichcandevelopplasticmoment
web criPPling under concentrated
resistancebuthaveinadequateplastichinge
,oiu,io" capacity for formation of a
plastic should be checked'
compact
..rrruoirr"befoie buckling are called as
6. Bending (Flexural) strength:
secuon
(b) ComPact sectron . Bending strength design oi laterall
(a) Plastic cross section
supportid beam is governed. by yie
(c) Semi comPact section (d) slender section ttr.ss and lateral or torsional buckli

develop plastic controls the design of lateral


Sol: Compact sections, which can
inadequate plastic unsuPPorted beams'
moment oi resistance, but have
of plastic (a) Lateially supported beams (1"1
fring. rotation capacity for a formation ia
hierally against lateral
rrpp"t
mechanism
torsional buckling)
Factored design moment at any
sectr
.
itti s Desigulending strength of secti
Which one of the following curve -represents (Md) ,,'
moment curvature relationship of . Unt; dt* < 67 e (No shear buckling
'slender
is
;;;ii;"' shown in the given figure below web)
Nominal shear caPacitY (V") -
R6{att*n&l fiY

a tT,
a

suscePtible to shear buckling)

Case I: Low shear case - (Factored de


shear force VS0.6 Va)
"^--
b.tign bending strength Ma: 9t Z,ftl y
1.2 Z"fyl T-o (For simply supported
eurualuft 0 =

(a) Curve 'A' (b) Curve 'B' <1.5 Z"f/T,no (For cantilever beams)
icj C.true'C' (d) Curve'D' Where.
\
I
Sol: Slender sections in which elements fu: 1.0 (for Plastic and comPact
buckle locally even before attaining
of yield sectlons
,ar... ur. .u11.d as slender sections' Curve 9r-- Z" lZn (fot semi
compact sections)
'A' represents moment curve relation of Z" and Ze elastic and Plastic
slender section.
modulus of the cross section
5. Design Criteria of a beam: For slender sections
o Design for bending moment Ma: Z"f, (fr- Reduced design strength)
o Design for shear force
ACE
Unnginccriogecaaemv :81 : Beams

Case II: High shear case - (Factored design 7n : Plastic section modulus
shear force V > 0.6 Va) f.6: Design bending compressive stress
Design bending strength
Mo: Mau : tu
M6u - design bending strength
for,--:-
Imo
under high shear f,.'.
: Bending stress reduction factor to
account for lateral torsional bucklins
For plastic or compact section:
1
t'2z"rv Tu: < 1.0
p(no, -n4,)= where B=f+-,)' 0,,, + (Oi, - r',, )o'
Y'o [v, )
M,1: Plastic design moment of the whole 0rr : 0.5[1 * crrr(Irr -0.2) + ),"2Lr]
section neglecting high shear case and
ctrr : Imperfection factor
considering web buckling effect
V , Factored applied shear force :0.2I (For rolled sections)
V,1 = Design shear strength as governed by :0.49 (Welded section)
web yielding (or) web buckling. Irr: Non dimensional slenderness ratio
M1,1- Plastic design strength of area of cross
section excluding shear area. JBZ;$<[.hqtM",
l,'or Semi compact section:
Mau : Z" frly^o
1lr) l.aterally unsupported beams (For beam
runsupported laterally against lateral
torsional buckling) M".:
. lleam with major axis bending and
compression flange not restrained against : Ba'Zr.f"r.1o
lrrteral bending fail by Iateral. Torsional M"r:the moment at which a beam fail by
buckling before attaining their bending lateral buckling when subiected to
slrength
uniform moment is called elastic critical
. 'l'he effect of lateral torsional
buckling moment.
rrocd not be considered when l.rr < 0.4
(where lrr - non dimensional effective
slunderness ratio for lateral torsional An unstiffened beam of ISMB 400 is subjected to
lrrrckling) transverse loading. I section is free from shear
Thc bending strength of laterally unsupported and torsional-flexural bucking. The yield and
boam is given by ultimate tensile strength of steel are 250 Mpa and
410 MPa respectively. The partial safety factor
Mo - Fu.Zp.f,a against yielding and ultimate tensile strength are
' Fb - 1.0 (For plastic and compact sections) T-o:1.10 and y,nr:l.25 respectively. Thickness
- Zclzp (for semi compact sections) of web is 7.5 mm and thickness flange is
I2.4mm, A:7840 mrr]; 2"":1020x103 mm3;
lr - Elastic section modulus
Steel
ACE :82:
nngioceriogAcaftqY
. Load Parallel to width O) A":
Z^;-1176.18x103
"#pu"irv
mm'' The design flexural
o,il section about its major axis is

(a) 33a.50 kN-m (b) 385-


19
kN-m
iri zg r.18 kN-m (d) 267 '31 kN-m
s
:250N/mm2'
Sol: f" :410 N/mm2 ; f,

Tmr: 1 .25 &T-o: 1'10


76' 1 8x 1 03 mm3
2",:1020x!03 mm3 ; Zr":!1 aud RoltedChalnels

When beam is free from flexural-torsional


buckling and free from web buckling is subjected
An unstiffened web of ISMB 300
t
Design flexural strength of an I-section is t *tu.tt. loading. The web of I- sectionultii.:
;d"i to tully plastic homent capacity of
a il; ,tt.t bucking' The Yield -and
section tensile strength of steel ate .250 YPu,ta
safety
+iO rtrp" reslpectively' The nartill
ugulou yielding and ultimatt T"tll:
stlfeth
176' 1 8x103x 250/1' 0
yl":i.ro and y;-r:l'25 respectively' Thickn
1
M6 = Zp' frl \^o: 1

:267 '3'7x 106 N-mm :267 '37 kN-m .'i" .*,"h is 7.5 mm and thickness flange

1. Shear strength of laterally supported


beam: Sol: - 1 rn.
\n: l'25
Design shear strength of section f":410 N/mm2; fr:250N/mm2'
Va: Au E*/ J3 t-" T-o: 1'10

Au - shear area t*:7.5mm andh:400mm


mm2
fi* - YielA strength of
web Shear Area: A':400 x 7'5:3000

Shear area Au Design shear strength of section

Va: fr-/ 3000 x2501 rE "t't


^6 T-o:
For l-section and charurel section Au
a)
. Major axis bending: :393.63 x 103 N:393'63 kN
Hot rolled - An: h t*
Welded-Ao:dt* Deflection Limit:
. Minor axis bending: . E*r.rrive deflections may lead to crac
Hot rolled or welded- An: 2b tr
lhe ptaster or ceilings and may
duT?gt
(or) supported by
-ua.tiuf afiached
b) For rectangular hallow sections of beam. This limits the
uniform thickness deflection.
. i""a parallel to dtPft (h) A": MO*h)
ilt*ffi.*= :83: Beams

\b(b+h . Other reasons for limiting the deflections Stress concentration occurs at thejunction
are of the web and the flange. As a result
(a) Excessive deflection may create large bearing are developed joint below
problems for roof drainage the concentrated loads consequently the
(b) These may cause undesirable twisting web near the portion of stress
and distortion of connections and concentration tends to fold over the
connected members and lead to high flange. This type of local buckling
secondary stresses. phenomena iscalled crippling or
. Deflections can be reduced by increasing crimpling of the web.
the depth of beam section, reducing the Web crippling is buckling of web caused
span, providing greater end restraints. by the compressive force delivered
l,imiting deflection various beams as per through flange to keep the bearing stresses
1S800:2007 is given below within permissible limits, the concentrated
. Vertical deflection for simply span load should be transferred flange. To the
rjected Elastic cladding - span/240 web sufficiently large bearing area.
rn is Brittle cladding - span /300
ulti . Vertical deflection for cantilever span
IPa Elastic cladding - spanll2D
:ty fac Brittle cladding - span /150
ength . Vertical deflection for purlins and grit
thic Elastic cladding - span/150
flange Brittle cladding - span /180
section

A (rm simply supported floor beam of an


:1.25 Itrrlustrial building supported by a concrete floor
hrrrcl. The maximum deflections as per IS 800,
In)50.00 mm (b) 25.00 mm Elf&lt t *iilh alaut bsrisE

(e )20.00 mm (d) 33.33mm


A''f-
The bearing strength is f*:
* lftrl:
y_o

Nltrrn of the beam L:6m:6000


mm f*: yield design strength of the web
l,irniting deflection of simply supported beam Ar: effective area of the web : brt*
5 x1.1 rrrpported by brittle cladding as per 15800:2007 is
ryrrrr/300 The angle of dispersion of the load is
KN
|,i rniting deflection: 6000/300:20mm assumed to be l: 2.5
Bearing length br : b * 2n1 (under
r crack 0. Web crippling: concentrated load)
rmage . Loads and reactions concentrated along a
: b * n1 (under reaction at support)
br
dbv short length offlange ofbeam are resisted
naxl by compressive stresses in the web which n1:2.5 (tr+ Rl)
vary with distance from the load.
.efi"" AcE
:.fDiEndne€fogacademv :84: Steel
w-:-

11. Built uP beams (Plated Beams)


-
. The bearing strength calculated should be
. Used when long sPans and heavY
greater than the concentrated load (it will
large B.M are generated when avai
safe for buckling)
. To eliminate web crippling, bearing rolled beam sections don't Pr(

stiffeners are to be Provided'


sufficient strength to resist the
. B.M.
To provide thicker webs
. When the dePth of the beam maY
restricted due to head room requirement
10. Web buckling:
. The web in rolled steel section behaves
like a column when Placed under
concentrated loads. The web is await thin
therefore subjected to buckling
Web buckling (vertical buckling) occur
when intensity of vertical compressive Buill-uP beam
stress near centre of section becomes
greater than for Iu: Ip r"q -I (Divide bY'hl2')
acti
the web mg
Io Io*o
of column the -
restraints provided for the flanges'
hl2 hl2 hl2
Vertical buckling of web is not a problem Zu:Zp'r"q-Zp'
with rolled beam sections, this possibility
exists in the thin webs of deep plate girder Zu: Au.h ? Au : ZJh
The maximum diagonal comPression h: Overall depth of I - section
occurs at NA will be inclined to 45" to it
The web buckling strength of support will Zr:Plasticmodulus of I - section: M/&
be M : factored bending moment
Fwb: B.t*.f"o Zu:Plastic section modulus of plates
required
Ip req: Plastic moment of inertia of
section to resist the total
moment
I: Moment of inertia of the desirable
dispersion 45" to the level of NA beam section available
Io: Additional moment of inertia of the
section required from plate section
M.I of plates to be calculated
rr$isr+j*!L:: Au: Bx t6.
B: Width of flange coverPlate
trp : Thickness of flange cover Plate
M..: Elastic critical moment corresponding
lateral-torsional buckling of beam
Eficcr,w..ffrh for wcb Oueftllry

lTirupati lGunu'rl
HyderabadlDelhilBhopallPrmelBhubmeswarlBengalmllucknowlPatnalcherrnilvijavawadalvizg
:85: Beams

Permissible or allowable bending stress


loads, Working stress method compressive or tensile stress
ailable o6, (or) o61 0.66 : t
rovide ll. l)csign Criteria of a beam: Where t:
yield stress of steel
rternal r l)osign for bending moment
r l)csign for shear force (b) For laterally unstrained beams (when
ay r ('hcck for deflection compression flange is unrestrained laterally
rent r ('heck for secondary failures like local against buckling)
lruckling of compression flange or web, web
crippling under concentrated loads should be o Permissible tensile bending stress
crlrccked. o61:0.66 t.
I l,
l)csign for bending moment: o Allowable compressive bending stress
l'he actual bending stress obc,cal (o6r) for laterally unrestrained beams
;r urrrllressive) afld obt,car (tensile) are dependson yields stress, lateral
r rrh'rrlated support condition of the beam and

ok.. or ok
MM
: _ \/ - _
slenderness ratio
0.66frf"r
.,"ealutcal
IZ ob" = < 0.66fy
M : Maximum bending moment [(fr)" +G")"1%
| - Moment of inertia of the cross section of Where,
the beam about bending axis f.r: Elastic critical stress in compression
y = distance from neutral axis to the extreme
lft compression or tension fibers
Z = modulus of the section: (I / Y)
EI: Flexural rigidity of the cross section
ll'allowable bending stress, o6 is given, the o The o6s and ma] be increased by 33
o-61
'Z' required is found as follows ll3 %o when effects of wind. seismic
Zreg M / ou. forces are taken into account.

le ll' 'Z'avallable and o,6 ore given, then the 14. Design for shear force:
r rroment of resistance, The shear stress at any point on the cross
the Mn: ebc .Z section of beam is given by t =
Va.Y
)n l.b
Pcrmissible bending stresses: V: shear force at the section
I:moment of inertia of the section
(n) For laterally restrained beams: (When b :
width of section at the point where
compression flange is restrained laterally shear stress is calculated
against buckling). a.y: Moment of area above the level where
the shear stress is calculated, about
neutral axis of the section.

Hydenbad I Delhi I Bhopal I Pme j Bhubmesw I


Bensaluru | Imknow I Pann I Chemi Vijayamda Vizas Tirupali Guntr I Kukrp{ly(Hyd)
I i I I
sffi.. AcE
:!4Eif"Ctt*tinsecademv
_:- :86: Steel

. Maximum shear stress occurs at neutral


o Web buckling occurs due to col
axis.
action of web under concentrated
o and due to diagonal compression due
Calculated Average shear stress
shear.
V
'vacal 17. Built-UP Beams:
h.t *
o Used when the available rolled
h: Overall dePth of beam, is insufficient and dePth beam of
t*: Thickness of web restricted.
o Approximate area of each cover P
required, An
o Permissible shear stresses:
Permissible maximum shear stress Ae
tu-: 0.45 t Where,
Permissible average shear stress tuu:0'40 t
Z.: Required section modulus
: M/o6.
n The permissible shear stresses may - be
increaied by 33 ll3ohwhen effects of wind or Zu: avallable section modulus,
seismic forces are considered' h: depth of beam section
o The gross area of flange plate is taken
15. Check for deflection:
20o/o of extra to account for rivet holes'
o The maximum deflection of a beam should
not exceed 11325 of the span in general' The . The outstand of flange plate (beyond
limit may be exceeded in case where the rivet should not exceed l6t
grater deflections does not affect the strength
compression) and 20 t for tension'
or crack the floor finishing
If welded built uP beam
o Permissible denection j?'d:Hl'JilT o Shear force at the weld Per I mm I
(horizontal)

o permissible denecti:ilrJiil5:Hl;Itt q=va.y


r
ay :
First moment of area about NA'
For riveted built up beams, the actual
bending stresses are calculated as follows
Calculated bending compressive stresses
MM
ob."", =;-'Y = -
'-rZ
Load dispersion under concentrated load
is assumed to be 30o with horizontal, for Calculated bending tensile stresses
computation of bearing stress at roof of An
obt"u, = ob""u, X-
frllet. An.,
Web crippling occurs at a section where
there is more bearing stress at the root of
Where Ar: gross area of the flange
Anet: net area of the flange
the fillet.
rctures 287: llcams

uflrnar
- Class Room Practice Questions
(c) A compression flange of hcnrrr is
loads restrained laterally
due (d) A web of beam is strengthened
Which one of the following cross section can
rlcvelop plastic hinges and have rotation 06. An angle is connected to the back <ll' llrr.
capacity required for failure of section by flange of a channel section to be used at rr
Iormation of plastic mechanism are classified beam as shown in the diagram below. This h
ls per IS 800-2007 done to
(a) Slender section
(b) Semi compact section
(r;) Compact section
(d) Plastic section
(a) Increase the compression flange area
Which one of the following cross section in (b) Increase the moment of inertia about the
which elements buckle locally even before major axis
irttaining of yield stress are classified as per (c) Increase the moment of inertia about the
rs 800-2007 minor axis
(r) Slender section (d) Make the load pass through the shear
(b) Semi compact section center
(c) Compact section
ken (d) Plastic section
es.
07. Web buckling occurs in a beam due to
excessive
l)esign ofbeam governed by shear
rnd (a) Direct tensile stress in the web
(a) When depth of beam section is small and
rt beam is loaded uniformly (b) Bending tensile stress in the web
(b) When large concentrated loads are placed (c) Torsional shear stress in the web
near beam supports
(d) Compressive stress in the web
(c) Both (a) and (b)
(d) None of the above is correct
08. The semi-compact section of a laterally
ll,l. 'lhe beam section is classed as low shear case unsupported steel beam has an elastic section
when the factored shear force as per limit modulus, plastic section modulus and design
state design of 15800:2007 is bending- compressive stress of 500 cffi3,
(a) < 0.6 V6 (b) s 0.6 Va 650 cm' and, 200 MPa, respectively. The
(c) > 0.6 Vc (d) > 0.6 vd design flexural capacity (expressed in
kNm) of the section is
ll1, A beam is called laterally restrained beam,
liom structural point of view when
(a) A web of beam is stiffened with 09. An unstiffened web of ISMB 500 is subjected
transverse stiffeners to transverse loading. The web of I section is
(b) A tension flange of beam is restrained free from shear bucking. The yield and
laterally ultimate tensile shength of steel are 250 MPn
and 410 MPa respectively. The partial salbty
factor against yielding and ultimatc tcrrsilr:
HvderabadlDelhilBhopallPmelBhubmeswiBengalmlLucknowlPana Chemil\tjayamdalMzg lTirupari lGunnrrj Kuhtpully(llyrl)
T AGE
YEngimlngAcademY
Steel Structures

strength are T.o:1'10 uttg . T^ft'25


i.tp.l.Uu.ty. Thickness of web is 10'2mm 01. (d) 02. (d) 03. (c) 04' (b) 05' (c)
und tnl.m.ss flange is 17'2mm'
the design
I section is ld{ 06. (d) 07. (d) 0s.(100) 0e.(66e'2) 10' (b)
shear caPacitY of

10.

ed

load
(c) Excessive bending moment
ins moment

t
I

: :: - nailviiavawadalvize lrirupati lGun*l


HvderabadlDelhilnnoo*1r*"1-
Plate Girders
.c)

10. (b)
01, Inlroduction: . Splices
A lirbricated plate girder is employed for

Limit state Desisn C

03. Economical depth of plate girder


d: [M Wfr],,' (wher.e k _ d/t*)
il
Where, d - Depth of the web palter
i-".r t* - thickness of web
3*tio A-A

o llsc two or more regularly available 04. If d/t* S 67 t*; (may be designed as ordinary
scctions, side-by-side. (may not satisfu
tlc llection limitation) beam) fwhere e*:
r llso a fabricated plate girder, wherein the
tlcsigner has the freedom (within limits) to t* - yield stress of web
t'lroose the size of web and flanges, or
r 05. Minimum thickness of web plate:
llsc a steel truss.
Minimum web plate thickness should meet
'\ plrto girder is a sglviceability and compression flange
buckling criterion.
r ltuilt-up beam fabricated from rolled steel
lirtes
Q Minimum web plate thickness based on
;r
s!rvrcggbillry requ i reme nr
r l,irrgely provided in bridges and also in
(a)When transverse stiffeners are not
lrrrildings for long spans with heavy loads.
provided,
r l)lirte girders are economical when span is . d/t*
lrrrge, say beyond 20m and the loads are
flanges along both longitudinal edges)
Irclvy. . d/t* < 90 e (Web connected to flanses
lrllt'rrrcnts of plate girder:
along one longitudinal edge only)
. Wcb plate
r lilrurge angles with or without flanse (b) When transverse stiffeners are provided,
plirtcs for bolted/riveted plate girder. . dlt*<200e* for3d>c>d
r Strl'l'eners
. clt*< 200 s* for 0.74d< c < d
> Vertical or transverse or stabilitv . dlt* < 270 e* for c < 0.74 d,
stiffeners (C-spacing of transverse stiffener)
> lJorizontal or longitudinal stiffeners
> I-oad bearing stiffeners (c) When transverse stiffeners
> ljnd bearing stiffeners longitudinal stiffeners at one level only
and

provided (0.2d from compression flange)


. dlt*<250 e* for 2.4d> c > d
/(
Steel Structures
:90 :

to flanges along
d/t* S 200 e (Web connected
. c/t*5250e* for0'74dSc<d both longitudinal edges)
. d/; s 340 e* for c < 0'74 d
and one t*Z d/200e
one at 0.2 d
and second
Jzso 'J250
-1n
"-= fy 250

. d/t* 5 400 e* where t*= @


1t.
""'
06. Minimum web thickness based

,otpt.tsion flange buckling requirement


are not
(a) When fiansverse stiffeners
Provided, ---
. t*d 3345ef2[where "= 'rl@/
\i[vr
are provided'
(b) When transverse stiffeners

. ds3a5*
Stilfenem
forcZl'5d LongitudiMl Stiflenec

t...

. d=345er forc<1'5d
t*

In a welded Plate girder,


to flange along both 1 stffiners
,ninimum thickness of w . Longituiliaul Mil Trailsvrse

;teel is required bY c Ansle sections are riveted or


criteria for un-stiffened tffi;; plate gird or Plate
of the web Plate is 2400mm section for welded Pl
(a) 12.0 mm @) 10'0 mm
it".f* tntermedi rs dePends
i.) ze.O ** (d) 6'0 mm "f
on thickness of web
f'=119ll*t' To avoid local buckling of tranwerse
Sol: For Fe410 grade- plates'
r'-----'t,
=zso N/mm2 ;iff;t,- outstand from face of web of
tit"tiJti* exceed2 hickness

stiffener)
Depth of Plate girder d=2400mm Minimum moment of inertia
of transverse
Minimumthicknessofwebplatebyconsidering stiffener
serviceabilitY criteria
Acadcmy :91 : Plate Girden

If cld>-Ji I,>0.75dt*3 o Connection between llange plates arrtl


tf cld< Ji Ir> 1.5 d3 t*3 I c2 flange angles is designed for horizontal
shear force.
r Intermediate stiffener not subjected to Connection between flange angle and web
external loading should connected to the plate is designed for horizontal shear force
web so as withstand to shear (in klr{/mm) and vertical shear. Here rivets will be
of not less than :*2l5 b, kN/mm under double shear.
(br--outstand of stiffener) No. of flange plates in each flange should
I r ) t,ongitudinal (Hori zontal) stiffener not be more than three.
Longitudinal stiffener increases buckling
resistance of web against@llq
These are provided between transverse
A welded plate girder is provided with various
stiffener
Minimum moment of inertia of first
stiffeners, if
depth of web plate is 2000mm
x 10mm, spacing of vertical stiffener is 1000 mm,
fongitudinal stiffener provided at 0.2d
then minimum moment of inertia required for
liom compression flange I > 4c t*3
transverse stiffener is
Minimum moment of inertia of second
longitudinal stiffener provided at N.A
(a) 200 x lOa mma
(b) g00 x 10a mma
lz dz t*3
(c) 1200 x 104 mma
(d) 600 x10a mma
(dz-2 x distance from compression Sol: Depth of web plate: d:2000 mm
lange to N.A) Spacing of transverse stiffener: c l000mm :
elt splice: a Z\,tot- cld:100012000:l12 <J,
It is a joint in the web plate to increase the Minimum moment of inertia of transverse
lcngth. stiffener
Wcb Splices should not be located at Ir> 1.5 dt t*t I c'
rcctions where shear force
oc0urs. >1.5x (2000)3x(10)3/10002: 1200 xl0a mma
Wcb splices are designed to resist shear
gltd moment at the spliced locat
llplice plates are provided on each side of Working Stress Method Concepts
llrc web plate for riveted connection. In
wr:ldcd construction, splice are made 10. Elements of riveted plate girder:
prclbrably by complete penetration butt . Web plate
welding. . Flange angles with or without flange
Fgc splice: plates for bolted/riveted plate girder.
It is n joint plate to increase . Stiffeners
lerrgth of flange plate.
splice should not be located at the stiffeners
of maximum bending moment
;92: Steel

Splices
14. In welded connection, the effective
area is equal to that of the flange plates
tongue plates, if any upto eight times
th
thickness

15.In riveted plate girder, the flange set


consists of flange plates + flange angles
part of web which is between the
angles is called web equivalent'

16. In designing the total flange area, the


fla
angles tttutt U. kept to have more u:':
in.
flaige, preferably at least equal to 1/3 of
area of tension flange.
Approximate self weight of plate girdet
(w)
11.
kN per meter run
17. Web Equivalent: The Portion of web w
* = W:OO (For riveted or bolted Plate acts ut flung. is termed as 'Web equiva
girder)
the web equivalent while calculating
W:Total load on the girder in lO{
area of flange is taken as 116 of web
area

girder (D): for net ur.uLf flunge is taken as 1/8 web a


12. Economical depth of plate
(The depth at which weight of girder is o Gross area of web equivalent: .4*/6
minimum) c Net web equivalent: A*/8
[ M o As per IS 80q,.- 1984, web equivaler
-Q
For riveted Plate girder, D = 1't.l
-
\13'-
thaf portion of 'iieb, which is between
flange angles'
IM
:
For welded Plate girder, D = 5'3.,f 18. Web stiffeners:
\loo
IS 800:1984 recommends to provide web
Where, M = Maximum bending moment' stiffeners as follows:
or = Permissible bending stress
o It depth of girder is less than 750mm' it is {= l.rr.rorpunaffand8s
called shallow Plate girder'
t* {ruu,"ot Jft
o It depth is more than 750mm, it is called
deeP Plate girder' No stiffener is required

13. Minimum thickness of web plate (t"): d'-<


(ServiceabilitY criteria)'
l.rspr6lpand2oo
tw { Tuu,.ut
. t*: 6mm (when Plate is exPosed to
Vertical stiffeners are Provided
weather but accessible for painting)
. t*= 8mm (when web Plate exPosed to 9= 9and25o
l.rr.rof
weather but is inaccessible for t* {Tuu,"ul
I
Painting) Vertical stiffener + first
I
stiffener at a distance from

I
:93: Plate Girdsrr

fl flange equal to two-fifth of distance from l5oMD'


llates compression flange to neutral axis Increaseof , - .*+
Et*
ES
. d' . l.rr.rof gand4oo o Lateral loading on stiffener
t* {T uu,.ur o'3v D'
Vertical stiffener + first horizontal Increase of I = .-+
Et*
stiffener at a distance from compression
flange equal to two-fifth of distance from
M - Applied bending moment (kN-m)
compression flange to neutral axis .| a
D - Overall depth of girder (nrm)
horizontal stiffener at neutral axis E - Young's modulus (2 xl0'MPa)
t* - Thickness of web
dz:2 x clear distance from compression V-Transverse force taken by stiffener
)a ln (ld'I)
flange angle or plate to the neutral
3of
axis.
(b) Longitudinal (Horizontal) stiffener:
. In no case the greater clear dimension of a
. Longitudinal stiffener increases buckling
b resistance of web against bending
web panel less than 270 t* and the lesser
rival
clear dimension of the web panel less than
. These are provided between transverse
ro
.D stiffener
180 t*
area . Minimum moment of inertia of first
eb longitudinal stiffener provided at 0.2d
lilf l'rilnsverse (stability or vertical) stiffener:
,16
. from compression flange I > 4c t3
lntermediate transverse stiffener increases
buckling resistance of web against shear
. Minimum moment of inertia of second
V
. longitudinal stiffener provided at N.A
Angle sections are provided for riveted or
veen
bolted plate girders and flat or plate I> dz t'
section for welded plate girders dz - Z x distance from compression flange
. Minimum moment of inertia of transverse to N.A
stiffener
I> 1.5 d,13 t3 I c2 19. Intermediate stiffener (vertical and horizontal
t = Minimum required thickness of web
stiffeners) are connected to web by rivets are
c = the maximum permitted clear distance
welds to withstand a shear force between
between vertical stiffener for thickness t*
web and each component of the stiffener of
r Maximum spacing of vertical stiffeners -
not less than 125 t*"15h (kN/m) (h--outstand
1.5dr.
of stiffener in mm)
r'r t Minimum spacing of vertical stiffness -
20. Load or End bearing stiffener:
,, | 0.33dr.
o Bearing stiffeners provided at the points
lf vertical stiffeners are subjected ti
of concentrated loads and at support to
cxternal force, the moment of inertia
resist reactions.
should be increased as
r Bending moment on stiffener due to
o Provided to avoid local bending failure of
cccenhicity of vertical loading with the flange and local crippling and
buckling of web.
rcspect to vertical axis of web
o Bearing stiffener should not be joggled.
Acad€my
:94:

area between
Must have suffrcient contact in
f;;;.'il the stiffener to transfer load
01. Welded Plate girder
weight less than
plate girder bY about
(a)0-10% (b) 10 % -20%
(d) 15 % -2s%
@)5%-15%
with web the angle sectton
The bearing stiffener together 02. In bolted plate girder'
iiui. tnuil be designed as a column' should be
length : 0'7 x actual length
'Effective of
(a) Equal angle
longer leg
stiffener.
centre line of
iil uo.q"ur angle section with
it. ,'n," is calculated about horizontal
leg
web Plate. (c) Unequal angle with shorter
Th.' ur"u of section which resists (d) A bulb angle
+ area of
,o*pr.rrion is area of stiffener
;;"f-;iength of 2ot* on both sides of 03. Economical (optimum)
depth of plate
the centre line of the stiffener' concePt based on
provide restraint (a) Minimum width
Bearing stiffeners are to
at the
Iguirr.,-,"tri"n of the plate girder iui tvtini*um dePth
ends ic) Minimum wight
ot weo
D'T id) trtini*rrm thickness
r=
250 w "* about 04. Intermediate vertical
stiffeners are prov
I - Moment of inertia of stiffener plate girder to
centre of web -
web buckling
tu) f,itnlttute shear
D - Overall dePth of girder buckling
i- Mu*imnm thickness of the iuj Etmittate local
concentrated loads
comPression flange
i.j rtuntrtr
id) lrevent excessive deflections
W-Totai load on the girder between
suPPorts
05. Longitudinal or horizontal stiffen'

from web Ptouid.d in Plate girder to 1


21. The outstand of all stiffeners buckling-
11,-
-lh: t for shear web
(16 iuierirni"u,.
\*, -"---
should not be more than 25(i-'lf v \- / Eliminate co
(b)
web

steel sections) and l2t for


flat sections (c) Transfer concenff atel lo19s
deflections
t - thickness of section or flat id-) Prevent excessive
:95 : Plate Girders

Which of the following statements are 10. Web and flange splice sh be located at
corrcct? the point respectively are
(l) l, II and III (b) II and III (a) Minimum shear force and minimum
(c) l, il and IV (d) I, II and IV bending moment
(b) Maximum shear force and minimum
% (l/, ('onsider the following statements: bending moment
% llorizontal stiffener is provided when (c) Maximum shear force and maximum
l. The depth of webs is small bending moment.
2. Tendency to web buckling is less (d) Minimum bending moment and minimum
l. Vertical stiffeners becomes too close shear force.
4. Only thin plates are available for web
:r leg Which of these statements are correct?
(a) 1 and 2 (b) 3 and 4
(c)l,2and4 01. (c) 02. (b) 03. (c) 0a. (a) 0s. (b)
(d) 1,3 and4
06. (b)-'07. (b) 0s. (c) 0e. (c) 10. (a)
lll{ lonsider the following
( statements:
llearing stiffeners, in plate girders, are
l. Provided at supports
2. Provided under concentrated loads
.1. Provided alternately on the web
Which of these statements is/are correct?
(a) 1 only (b) 2 only
(c) 1 and2 (d) 1,2 and3

lI). In a steel plate girder, the web plate is


connected to the flange plates by fillet
welding. The size of fillet welds is designed
to resist.
(a) The bending stresses in the flanges
(b) The vertical shear force at the section
(c) The horizontal shear forces between the
flanges and the web plate
(d) The forces causing buckling in the web
il
Gsntry Girders
.1,

1. irders: These are


beams Provided in
buildings to carry cranes'
trougitndinal Loads
Vertical Loads
Eorirostal or
bails
. Lateral
Verticd Loads

Wheel Base

I;3Par ol GutrY Girder


GattrY Girder

Loerts Acting on GentrY


Girder

(
movmg over
The wheels of crane girder
ertical loads to gantry
1 loads consists of
Self
Troffy eht of crab or trolleY
and self-weight of
gantry girder etc
due to sudden
The lateral thrusts are caused
*;*; ;r starting of the crab and these
lorerql fnrces act normal to the rails'

8-8oob $*/
tl

i
I
Otgt^rg
,w ) lryurL fae )gor * Cg{^fo^i
F€ :-rt'

ACE
fl.ngineeringAcademy :97: Ganny Ginlcm

.1. Additional loads for structures subjected to


impact:

'l'ype of Load

(rr) Vertical loads transferred to the rails (


(l) For electric
overhead cranes
5. Limiting or Maximum Deflections:
(ll) For hand operated 10 % of maximum
The vertical deflection of a gantry girdcr
cranes static wheel load.
should not exceed the values specified below:
(h) l{orizontal loads transverse to the rails ( q(,Yg
Limiting
ll) Iior electric I0 % of the weight of or
overhead cranes the crab and the weieht maximum
lifted on the crane Deflection
lll) l"or hand operated 5 %o of the weight of (a) Where the cranes are
cranes the crab and the weight L/500
(b) Where the cranes are
g over lifted on the crane
overhead traveling and L/750
gantry
lr') l,ongitudinal loads operated electrically up
of Self
rlong the rails to 500 kN or 50ton
: trolley
ight
(c) Where the cranes are
for a Gantry Girder:
Section overhead traveling and
Some of the types of sections used for operated electrically
sudden
gantry girder shown below
6fl more than 500 kN or
d these
5Oton
rtopping (d) Other moving loads, such
produce as charging cars, battery
t gattry
Where L: Span of the gantry girder. C tvwn)
y girder
uced in
The compression flange of an.I - section
q*) is reinforced with a channel to increase
In an industrial building gantry girder of effective
span 22-.5m carries an. EOT carne of capacity ol'
400 kN. The maximum deflections as per IS tJ(X),
where 'L' is the span of gantry girder
(a) 16 mm (b) 20mm
(c) 24mm (d) 30 mm

HydembadlDelhi BhopallPuejBhubmewlBengalmlLucknowlParnlChemiiViiarawadaLvizs lTirupati lcunhrrl Xtt||t$dlt(lfid

,'1
))
'1lr
ACE :98 :
Eogine€ring4"4".[
3. Longitudinal loads
4. Wind loads
Select the correct answer using
the
Span of the beam L:22'5m=22500mm
supported by
Limiting deflection of Gantry girder siven below:
EOT cranes, crane capacity up to
500 kI'{ as per [a) 1 and 2 (b) 1,2 and3.
IS800:2007 is sPan/300 ici t ana 3 (d) 2,3 and4
Omm
Limiting deflection: 22500 l7 50=3 is used in
04. Assertion (A): Impact factor
design of a gantry girder'
n.uion (Ri Loads transferred to the
gan

girder are momentary'


Za) Both A and R are
true and R is the
correct exPlanation of A
(b) Both e and R are true but R is not a
correct exPlanation of A
(c) A is true but R is false
flange (d) A is false but R is true
'reduce
(d) To mpment of inertia about
horizontal axis 05" carr1 electricall

02. The given figure'shows a typical


section of a :il with heavY loads tt
.''i',i
.rurrJ girder. Consider the following eed
(a) L / s00 (b) L / 600
statements in this regard:
(c) L / 750 (d) L / 1000

06. Which one of the following statements


a channel
correct? In a crane ganIry girder'
provided aI' the top flange girder'
The function of the toP channel isto accounts for bending in
1. lncrease moment of inertia about vertical
axls
2. Reduce moment of
inertia about
horizontal axis
about
3. Decrease the slbnderness ratio twisting
minor axis
4. Increase lateral buckling strength
Which of these statements are correct?
01. (a) 02. (d) 03. (b) 0a' (a) 0s'
(
(a) 1 and4 (b) 2 and3- .

icj t, z and+ (d) 1,3 and4 06. (a)

Which of the following loads are to


be
03.
considered in designing a gantrY
girder in an

industrial building?
1. Gravity loads
2. Lateral loads
Roof Trusses
p4
l, All joints are assumed to be hinged and

All members of a roof trusses are subjected


kr axial forces tu)
T
h
fv \1 _ \.llr-
l4".'""*.dfuL*-*l
t
Slwtod Trnrs (5n toSn) ItleyfirighTruss ftpS I
Rise _h
t,ttch ot truss-_ -
Span L
2h
Slopeof truss= tan0 =*+ =
(Lt2) L
\Jpani HweTrusr (SA*ns6n
rical 2
rds t
Slope of truss:2 x Pitch of truss
Fink{or} Frcnch Trnre
Ffuk(orlFrench Trn*s (Ifu ie 2llo)
(spn<hl
I'he type of truss depends on span of truss
lnd pitch of truss. Pitch depends on type of
roofing material, ventilation and light F*n Trnss (Sgan < l?.m) CainpoundEla Truss (Sgu ?4 m b27u|
rcquirement:

. Small Pitch : < Ill2


Warren Truss Pntt Truss
. Medium Pitch: llI2to ll5 (Small pitch Spar)
& Lerge (iltediun pitch & Lrge Epr)

. Large Pitch : >ll5


6. Spacing of roof trusses:
. Warren truss is used for small pitch. al spacing of truss is Il3 to 1/5 of
span oftruss.
r Pratt truss is used for medium pitch.

. Firk truss is used for large pitch. For economical spacing,


Cost of trusses :2xcost of purlins * cost of
( lommon pitches
covering
for different roof sheets:
l'itch - 116 for GI Sheets

l'itch - lll2 for AC sheets


I'itch - ll4 for snow load with wind load
ACE : 100:
EndntcechgAcad€my

Common Rafter:
7. Elements of Roof truss:
These are provided only if the spacing
purlins are larger than the available I
sheeting. Rafters are inclined
suPPorting
of toP and
chords oftruss.
The reaction of the truss is transferred to
column through the shoe angle' I
supported on the base Plate'

Base Plate & Anchor Bolts: Base


supports the shoe angle of the tru-ss
bolts. Base plate is anchored to the
or wall through anchor bolts' Anchor
can take both downward as well as
reactions from the truss.
Bay: It is the distance between
trusses.
at nodal Points. Rise: It is the distance from the highest
o Struts:
The member carrying co forces in a
roof truss are called struts'
o Ties: Members carrying e forces'
o Main Tie (Principle Tie): The bottom chord
member. It is usually in tension and takes tangential components of the load' Genet
compression if reversal of loads occurs due
to twJtnes of sag rods are provided in each
wind load. which are connected to the ridge purlins'
o Ridge line and eves: . Lateral bracing of end trusses:
The"top line of the roof truss is called the required to resist horizontal loading (su
ridge line and the bottom edge of roof surface that due to wind etc).The Bracing for
is called eves. trusses and supporting columns provide
Purlins: rigid structure. Bracing is provided to
These are Members subjected to transverse tJo t ott.t on either side of the shed
loads and rest on the rafters of roof trusses' top chord and bottom chord levels' Sim
They support sheeting's that cgries roof the last two supporting columns at either
covetittg.'ih.y are horizontal beams are to be verticallY braced.
spanning between the trroo adjacent trusses' Bracings are not required if the
Spacing generally varies from 2m to 3m' supported on masonry walls and if s
Sag tie: To reduce gable walls are Provided.
Fu. to self weight,
:101: Roof Trusses

r 'l'lro bracings are provided in such a manner ii) For member supporting the roof members
tlrrt their diagonals form angles about 45o and roof purlins, such as trusses, beams,
with the load to be carried. girders etc,2l3 of load in (a).

c) Snow load: 25 N/m2 per one centimeter


ng depth of snow.
d
roof truss ifpitch k and slope t{, If e > 50o slope, snow load need not be
f tr rr
+
{3\i3
*re
considered.
red to
ntrglo of inclination with horizontal would be
.It 16f l0o (b) 45" (c) 60' (d) 75" d) Wind load: Design wind pressure,
V6 : basic wind speed in m/s at a height

iase
ftll: Slope of roof truss 10 m at the locality. It is to be obtained
from the code. The code divides the
iS lan0: Rise of trussAlalf soan:l
'lc country into six zones for wind velocity
te r'/ J
calculations.
chor
: I :30o
IS 0 tan-l t- V,: Design wind velocity in m/sec at a
{J height z
L adj
ll l)csign loads on roof trusses:
Vr: kr. kz . kl.Vu
5hest l)t:ad Load, Live Load, Wind Load and Snow k1 : probability or risk factor
I oads
'el l) Dead Load (DL): k2: terrain, height and structure size
ne r) Roof covering factor.
rf r ) Purlins weight
ied by K3 : topography factor.
rr) Self weight truss 100 to 150 N/m2 on plan
area P, : design wind pressure in N/m2 at a
n each
height, z
rlins.
Braci Where L: Span of the truss P.:0.6Y,2
rg(
Lg for b) lmposed or Live load (LL): The design wind pressure on a roof is
rovide l'or slopes < 10o determined by combination of external
Ito wind pressure and internal wind pressure.
hed
i) Live load is 1500 N/m2 if access is
. Simi
provided. (i) External wind pressure: It depends
either ii)Live load is 750 N/m2 if access is not on slope. The external wind pressurc
provided. in term of basic wind pressure 'p'
on roofs when wind is normal to ritlgc
IE lbr roofs sloping > 10o is as follows.
i) For roof membrane, sheets or purlins:
750 N/m2 less 20 N/m2 for everydegree (ii) Internal wind pressure: It clcpcrrrlr
increase in slope over 10o subjected to a upon permeability of the stnrt'lrrrr'
minimum of 400 N/m'. For different permeabilit.y nI
ACE : 102:
Enginc€dng4"a4"m[

buildings, the internal air pressure


in Sol: ofroof0:3Oo
SloPe
'P' is not
terms of basic wind presswe
is Imposed load on roof (when access
given below. provided & sloPe of roof >10o)

Type of Buildings )
: [750-20(30"-lu")l
N/m-

l.Zeto PermeabilitY, no openmgs :350 N/m2 not less than 400 N/


(Mriltistoried building with 400 N/m2
Minimum imposed load on roof is
I wall sand no
1- Normal PermeabilitY
(up to 5% oPeningl QtoY.of 9. Design of Purlin:
trusses
commonlY afforded bY . Purlins are beams provided over
"ii
structure through open support roofing between adjacent
Channels and angles sections are
c(
windows and doors
3. M"dintn oPenings used as Purlins
to
. Wirrd forces is assumed to act normal
4. LargeOPening ttos u"a gravitY loads Pass fu9"g1
(ArIa oi oPening > 20o/o of centre of gravity of the purlin
sectron

iotal wall area) (Hangers and ptrlin sections is subjected twtsttng'


to.
;tdi*" to bending such bending is cal

unsYmmetrical bending'
. p*iltt may be designed as a continuous
simply supported beam' IS 800 recollu
30m inN/m' tfre purtins to be designed.as
continuous
The self weight of truss of span
taken as subjected to bi-axial bending'
(a) 75 N/m2 (b) 1oo N/m'.
i.i tso Nl-t (d) aooo N/m" Design Procedure:
IS800:
att.-tt State Design Principles
Sol: The gravity load Pr due to self-weight
truss: L:30m purlin, sheeting and live load etc
Span of the
N/m2 Load due to wind Pressure Hr and
Self weight of truss: (Span/3+5)x10
: loads are multiplied by load factors
: (3013+ 5)xlQ 150 N/m2
P: Factored along minor Y-Y axis
: Yr(Pr cos0
+Hr) in kl'l/m

H :Factored load atong major Z-Z


axis

: yi.(Pr sin0) in lcN/m


: 103: Roof Tfumo

10. Design of purlin (Working Stress Method


r Maximum design bending moments Concepts, IS800: 1984):
. The gravity load Pr due to self-weight of
P(2 purlin, sheeting and live load etc
About major axis M,,: . Load due to wind pressure is H1
10
H(,2
P: Load along minor axis Y-Y axis
Ahout minor axis Mu.,
0N/ 10
: (P1cos e +Hr) in kN/m
,tf Y
**, H: Load along major Z-Z axis
/ Span of the purlin, i.e clc distance : (P1sin 0) in kN/m
between adjacent trusses or spacing of . Maximum bending moments
lruss

I'rlrlin are subjected to bi-axial bending and


About major axis Mr,: ry
l0^
tetluire trail and error method for their design
About minor axis M*, : Hl'
10
l)csign capacities of the section o The section chosen purlin must have extreme
. -Y f. f.. fiber stress with in the permissible bending
lv1.1, 'Zpr. < l.zz"r.:J- limits
T.o T,oo
f. f.. Mr,
-- __.
V+
M"r, M' Mr.,
" Z=-----==-+ "
Or, =O*^ =
M,lv = Zor.L 3yt.Z"r.L
T.o T.o
lr, ' I", Zn Zw
o The deflection in purlin should not be more
than permissible deflection
( ll' 'lr
'/.
t I .2 then y1 is used)

11. Deign of AIIGLE PURLIN conforming to


mlbty Mo, 2 M- (M"u) & Mdy > My, (M*)
t: steel grade Fe410-S and Fe410-O, Fe4l0-
ight l) 'l'he local capacity of the section is W with following specifications as per
IS800:1984:
olrccked using following interaction
o When slope of Roof not exceeding 30'
. M--
equatlon "'+ M*" <1.0 (e s 30")
' Md, Mau o Width of angle leg in the plane perpendicular

ll) 'l'ho deflection of purlin is calculated to the roof covering > -1


-45
which should be fess than deflection limit.
(Span of purlin / 180 for brinle cladding
o Width of angle leginthe plane parallel to the
ttnd span of purlin I
150 for elastic roof covering >
clntlding) ;
o Maximum bending moment in the purlin
w('"
M:
10
: 104: Steel

02. A channel section is placed in an inc


Where,
position carrying vertical loads as shown
w : uniformly distributed load Per
unit length on purlin including wind load
ihe given figure. If the applied T9-:nt.
the Jhannel is 'M' due to vertical load,
M** is given bY

Roof sbeeting

t"lfrtr
ror
it
I
1s) -M
I .,,

(d)2M

The bending moment in the purlin about


minor axis may be neglected and angle purlin 03. Z-purlins may be fixed in either orientation
or B as shown in the given figure' Which o
may be designed for above moment
of the following statements is correct in
regard?
/: t.e clc distance
Span of the purlin,
between adjacent trusses or spacing of
truss

Class Room Practice Questions


r_J
AB

(a) Orientation A is structurallY


efficient than orientation B.
(b) Orientation B is structurallY
in a roof trusses are efficient than orientation A'
01. Secondary stresses
(c) Both the orientations are struc
induced due to
equally efficient.
I. Purlins are place at intermediate pointson (d) Niither of ' the two orientations
panel length K:
': "
structurallY e fficient.
II. RigiditY ofjoints
III. Eccentricity of bolt line and centroid 04. Assertion (A): Latqal bracing of
of member section trusses in necessary in industrial buildings'
(a) I and III are correct Reason (R): Wind forces acting along
ridge line are to be resisted.
(b) I and II are correct
(c) II and III are correct (a) Both A and R are true and R is the
(d) I, II and III are correct explanation of A
Roof Trusses

lcli (lt) Both A andl{a,,,r,irt, l)ut tt is not a


)wn correct sxplaniltisll
ont fl (c) A is true but lt is
I rl A 09 - 7ye economic spacing of a roof truss depends
l,rln,,
c,
(rl) A is false but Il
is trrr,,

ll Which one of thc lolhrtt'tttli


is correct? The
prrllins in
.'sy :,;:n'"t"tj:fr'
(rr) is paralle
t'i1l:rl axis'but
.oesnt.
rs. perpendiculill
1,, rninor principal
iIX IS

is inclined 11,,,,,,,,,,,,
19 prirrci;'ral axis
('otnclde wlth thr,
rr rl r r'r prirrcipal axis
Ittttlirts are providrrl
.^^r
"',r,, ,,,
,,r ,,,ir,,.
ilt t1il h,,:rJ;-^-
r,.;,,r
ilrrrrlrrstlrlrl
tt il tsil buildings,
oul
rool r___- lu rl ty y
'v('l', rlcad
@) 02. (a) 03. (c) 0a. (a) 0s. (c)
'Lluo.)\,r
;,: . , .^;:' 'Ptt!* I rr rlcacl loads,
ilv('t()aOS
...,.,^, ..-^,r^^-'*-
"""1 ',,
ano \ryltlrl |lrttltl".
r

As pcr IS code, ol.


wlr:rt ure.they
96. @) 07. (4s") 08. (a) 0e. (a) 10. (d)
assurrrr',l ,
1,, ;,,,
lrrl Sirnp.ly supportt,rl
llrl (,rrrrtrlc
lil rlrlcv0f
lr )( ontinuous
lrll I'rrt'rl

lll Iilllgle
l.:::llT':. ll f,rrt, rr t,., ,rrrrt slope is 1,
lllt' ttl ttlcltnltll,,,,
rr illr lrorizrlrrtal would
I tr,

. v\
\,\ w ,,,t1'lllllll
I ||ilss _1'llr , illllliltl
l,lr I

i
!

llttllril ll||,,rilll l,rrrr ll[ilrlrrrr.rw,,, 1k.,,6,1.-


1 11[-o-r., iParnlChemai ivijarzwadalviag lTimpati lGunn[l KukaDalty(Hyd)

You might also like